Emergency Medicine History/Physical Exam

Réussis tes devoirs et examens dès maintenant avec Quizwiz!

One Step Further Question: Which alternative therapy has shown benefit in patients with anxiety and tinnitus?

Answer: Biofeedback.

One Step Further Question: What is the most common cause of hemoptysis?

Answer: Bronchitis.

One Step Further Question: What condition is the most common cause of stroke in young adults?

Answer: Carotid artery dissection.

One Step Further Question: What is the only FDA approved treatment of hiccups?

Answer: Chlorpromazine.

One Step Further Question: What is the main treatment for scombroid fish poisoning?

Answer: Antihistamines such as diphenhydramine.

One Step Further Question: What dermatome covers the nipple area?

Answer: T4.

One Step Further Question: What is the treatment for a cholesteatoma?

Answer: Congenital and acquired cholesteatomas can be eradicated from the temporal bone only by surgical resection.

One Step Further Question: What is the name of the terminal end of the spinal cord?

Answer: Conus medullaris

One Step Further Question: In addition to elevated ESR, what other lab abnormality is seen in 50% of patients with temporal arteritis?

Answer: Elevated liver function tests.

One Step Further Question: What is the most common cause of neonatal hemorrhage?

Answer: Failure to administer vitamin K in the immediate postpartum period (associated with home births).

One Step Further Question: What is the most common human enzyme defect?

Answer: Glucose-6-phosphate dehydrogenase (G6PD) deficiency.

One Step Further Question: In neutropenic fever patients with a bacterial infection, what bacteria are the most common source?

Answer: Gram positives.

One Step Further Question: What US state has the most cases of scombroid fish poisoning?

Answer: Hawaii, then Florida.

One Step Further Question: What tests are necessary if non-accidental trauma is suspected?

Answer: Head CT and skeletal survey.

One Step Further Question: What is the most infectious blood born pathogen?

Answer: Hepatitis B (followed by Hepatitis C, then HIV).

One Step Further Question: What are the components of the San Francisco syncope rule?

Answer: History of CHF, hematocrit <30%, abnormal ECG, dyspnea, systolic BP <90—defines high-risk criteria for patients with syncope.

One Step Further Question: How much vitamin K should be given in patients with active hemorrhage and elevated INR on warfarin?

Answer: They should be given 10 mg.

One Step Further Question: At what serum level after an acute salicylate ingestion is hemodialysis indicated?

Answer: 100 mg/dL.

One Step Further Question: What is the maximum dose of lidocaine without epinephrine?

Answer: 5 mg/kg.

One Step Further Question: What percent of abdominal aortic aneurysms can be palpated on physical exam?

Answer: 50%.

One Step Further Question: What GCS score is considered the threshold for intubation for airway protection?

Answer: 8.

One Step Further Question: What is the most common symptom of transport dysphagia?

Answer: A feeling that food is getting stuck after swallowing.

One Step Further Question: What is the Klein line in relation to SCFE?

Answer: A line drawn along the superior border of the femoral neck that should intersect the femoral head. If it does not, suspect SCFE.

One Step Further Question: What adjunctive therapy has been used to treat the symptoms of acute pharyngitis?

Answer: A single dose of dexamethasone has been shown to reduce inflammation and pain.

One Step Further Question: How is presbyopia different than myopia or hyperopia?

Answer: Abnormalities of corneal thickness and axial length of the eyeball are more commonly thought of as causes of hyperopia (farsightedness) and myopia (nearsightedness).

One Step Further Question: What is the most immediate life threatening complication of pituitary apoplexy?

Answer: Adrenal insufficiency.

One Step Further Question: What are the Centor criteria for streptococcal pharyngitis?

Answer: Age and history of fever, tonsillar exudates, tender anterior cervical adenopathy, absence of cough.

One Step Further Question: Which anatomical location does amyotrophic lateral sclerosis involve?

Answer: Anterior horn cells.

One Step Further Question: List some medications which cause altered level of consciousness?

Answer: Anticholinergics, anticonvulsants, antidepressants, antipsychotics, clonidine, illicits (cocaine, heroin, marijuana, ethanol, amphetamines), opioids, salicylates.

One Step Further Question: What class of medications is most commonly used to treat peripheral vertigo?

Answer: Antihistamines.

One Step Further Question: A cough that lasts longer than 6 days has been linked to what condition 25% of the time

Answer: B. pertussis, the causative agent of pertussis.

One Step Further Question: What is an example of a drug that reduces cardiac contractility?

Answer: Beta blockers.

One Step Further Question: What is the appropriate treatment for mitral valve prolapse?

Answer: Beta-blockers are used to treat atypical chest pain and arrhythmias, but no treatment is required for asymptomatic patients.

One Step Further Question: What history with subconjunctival hemorrhage would prompt investigation into a bleeding diathesis?

Answer: Bilateral or recurrent subconjunctival hemorrhage should have a bleeding diathesis workup.

One Step Further Question: What is the significance of "cells" and "flare" seen on slit lamp examination?

Answer: "Cells" and "flare" represent deep inflammation of the eye and is typically seen in iritis.

One Step Further Question: How long does it take for a subconjunctival hemorrhage to resolve?

Answer: 10-14 days.

One Step Further Question: Is Homan's sign a reliable examination finding in the evaluation of deep venous thrombosis?

Answer: Homan's sign is pain in the calf with dorsiflexion of the ankle. It is an insensitive and nonspecific sign of a deep venous thrombosis.

One Step Further Question: What hearing abnormality is associated with Bell's palsy?

Answer: Hyperaccusis.

One Step Further Question: Why should activated charcoal be held in patients with altered mental status?

Answer: In patients with altered mental status, the risk of aspiration is high and activated charcoal can cause a severe pneumonitis if it is aspirated.

One Step Further Question: What is Levine's sign?

Answer: It is when a patient describes their chest pain by holding their clenched fist over their chest. It is considered to be a sign of ischemic chest pain.

One Step Further Question: What is the significance of sacral sparing in a spinal injury?

Answer: It shows potential for recovery and is more common with incomplete cord syndromes.

One Step Further Question: What is the name of the toxin associated with tick paralysis?

Answer: Ixobotoxin.

One Step Further Question: What percentage of vertigo is due to central vestibular etiologies?

Answer: Less than 10%.

One Step Further Question: What imaging test is most helpful in the workup of vertebrobasilar insufficiency?

Answer: MRI of the brain.

One Step Further Question: What is the treatment for pertussis?

Answer: Macrolide antibiotics (e.g., erythromycin, clarithromycin, azithromycin). Prevention is achieved through administration of the pertussis vaccine DTaP in children < 7 years old, Tdap in children > 7 years and adults.

One Step Further Question: What are the most common sites of hemorrhage in massive hemoptysis?

Answer: Massive hemoptysis typically involves the bronchial or pulmonary arteries.

One Step Further Question: What is the most common X-ray finding in acute aortic dissection?

Answer: Mediastinal widening is seen in the majority of aortic dissection cases.

One Step Further Question: Which category (eye, verbal, or motor) of the GCS correlates best with outcomes?

Answer: Motor.

One Step Further Question: What is the most common cause of ventricular tachycardia?

Answer: Myocardial ischemia or infarct.

One Step Further Question: Which patients with altered mental status should receive empiric naloxone?

Answer: Naloxone is an opioid reversal agent and should be considered in patients who present with altered mental status with respiratory depression.

One Step Further Question: What gram stain scoring system provides a more sensitive and specific diagnosis than wet mount?

Answer: Nugent's score.

One Step Further Question: What organisms cause SBI in neonates?

Answer: Organisms responsible for SBI in neonates are those found in the vaginal canal: Group B Streptococcus, Escherichia coli, and Listeria monocytogenes.

One Step Further Question: What age groups are at higher risk for a malignant cause of their back pain?

Answer: Patients under 18 years or over 50 years of age.

One Step Further Question: In which patients should NSAIDs be avoided?

Answer: Patients with acid peptic ulcer disease, renal insufficiency, diabetes, CHF and a history of allergic reaction to NSAIDs.

One Step Further Question: What systemic disease is frequently associated with temporal arteritis?

Answer: Polymyalgia rheumatica (in 30-40% of patients).

One Step Further Question: What are the mechanisms of pulmonary bleeding?

Answer: Pulmonary hypertension, erosion into a blood vessel, and coagulopathy.

One Step Further Question: What diagnostic test can help identify right coronary artery involvement vs left circumflex involvement in an acute inferior wall myocardial infarction?

Answer: Right-sided electrocardiogram with a focus on V4R; an upright T wave in lead V4R indicates involvement of the distal right coronary artery (RCA), and ST segment elevation indicates proximal RCA lesion.

One Step Further Question: What critical diagnoses should be considered in patients with syncope and back pain?

Answer: Ruptured abdominal aortic aneurysm, aortic dissection, pulmonary embolism and ruptured gastric/duodenal ulcer.

One Step Further Question: Does LDH elevation have a greater sensitivity or specificity for Pneumocystis jiroveci pneumonia?

Answer: Serum LDH has a greater sensitivity (approximately 90%) in Pneumocystis jiroveci pneumonia but a poor specificity

One Step Further Question: What injury can occur if Hemoccult drops are instilled in the eye?

Answer: Severe alkali burns can occur.

One Step Further Question: What are the main concerns when reviewing an ECG in young patients with history of syncope?

Answer: Short PR, Long QT syndrome, Hypertrophic cardiomyopathy, Arrhythmogenic right ventricular dysplasia, and Brugada syndrome.

One Step Further Question: What is the treatment for infant botulism?

Answer: Supportive care and human botulism immunoglobulin (BabyBIG).

One Step Further Question: What spirochete infection is a common cause of sensory ataxia?

Answer: Tabes dorsalis (neurosyphilis).

One Step Further Question: What is the most common cause of headaches presenting to the Emergency Department?

Answer: Tension headache.

One Step Further Question: What is the discriminatory zone?

Answer: The discriminatory zone is the beta-hCG level at which a normally developing pregnancy should be seen. In general transvaginal ultrasound level is 1,000 - 2,000mIU/ml and transabdominal ultrasound level is 6,500mIU/ml.

One Step Further Question: How do NSAIDs relieve the pain of dysmenorrhea?

Answer: Their antiprostaglandin action counters the underlying pathophysiology.

One Step Further Question: What organism may be identified by a wet mount made from a vaginal swab?

Answer: Trichomonas vaginalis, a flagellated protozoan visualized on the wet mount causing an itchy, malodorous vaginal discharge.

One Step Further Question: What is the prodrome that is classically, but not always, associated with vasovagal syncope?

Answer: Vasovagal syncope is usually associated with a prodrome of nausea, warmth, pallor, lightheadedness, and/or diaphoresis.

One Step Further Question: What is the most common cause of acute unilateral lymphadenopathy?

Answer: Viral lymphadenopathy.

One Step Further Question: What is the most common gynecological condition in prepubertal children?

Answer: Vulvovaginitis.

What is the most common cause of minor hemoptysis in the emergency department? Acute bronchitis Pneumonia Pulmonary embolism Tuberculosis

Correct Answer ( A ) Explanation: Acute bronchitis is the most common cause of minor hemoptysis in the emergency department. Hemoptysis describes the presence of blood in sputum expectorated from the respiratory tract. The majority of episodes of hemoptysis are minor episodes with small amounts of blood. Less than 5% of patients with hemoptysis have life-threatening hemorrhage. Bronchitis results from inflammation in the bronchial tree and can lead to hemoptysis. Additionally, forceful coughing further irritates the inflamed mucosal surfaces and can result in bleeding. Hemoptysis in bronchitis is typically self-limited.

Which of the following conditions is associated with asymmetrical distal weakness without hard sensory findings on neuro exam? Amyotrophic lateral sclerosis Mononeuropathy multiplex Primary biliary cirrhosis Vitamin E deficiency

Correct Answer ( A ) Explanation: Amyotrophic lateral sclerosis (ALS) is the most common degenerative disease of the motor neuron system. Patients with ALS demonstrate asymmetrical distal weakness without sensory findings. ALS is a fatal disease, with median survival of 3-5 years. Aspiration pneumonia and medical complications of immobility contribute to morbidity in patients. Mononeuropathy multiplex (B) is characterized by an asymmetrical, sensorimotor, usually distal pattern of peripheral neuropathy. Sensory abnormalities tend to be located in the same general anatomic region as the accompanying motor findings. Primary biliary cirrhosis (C) and vitamin E deficiency (D) are both associated with sensory neuronopathies.

A 49-year-old woman presents to the ED with dizziness. She describes it as a sensation of spinning. It began several days ago and has progressively worsened. It is associated with nausea. Which additional finding would suggest a central rather than peripheral etiology of her symptoms? Dizziness is constant Horizontal nystagmus that fatigues Patient notes hearing loss Patient notes tinnitus

Correct Answer ( A ) Explanation: Central vertigo is caused by disorders affecting central structures such as the brainstem or cerebellum. The dizziness from central vertigo is constant, with a variable response to movement (can either remain unchanged or worsen). The symptoms are usually slower in onset and are often vague and ill-defined. Hearing loss (C) and tinnitus (D) may be seen with peripheral vertigo (especially in those with Ménière's disease) but do not occur with central vertigo. Nystagmus can be in any direction with peripheral vertigo (though horizontal or horizontorotary is most often seen) and fatigues promptly. In contrast, the nystagmus associated with central vertigo is usually vertical and (B) does not fatigue.

Which of the following best describes a congenital cholesteatoma? Benign tumor of the ear canal Fat (lipid) deposits deep in the edge of the cornea Localized deposits of fat that collects in the skin of the upper and lower eyelids Plaques that begin in coronary artery walls

Correct Answer ( A ) Explanation: Congenital cholesteatoma usually appears as a white, round, cystlike structure medial to an intact TM. Cysts are seen most commonly in the anterior-superior portion of the middle ear, although they can present in other locations and within the TM or in the skin of the ear canal. Affected children often have no prior history of otitis media (OM). Congenital or acquired cholesteatoma should be suspected when deep retraction pockets, keratin debris, chronic drainage, aural granulation tissue, or a mass behind or involving the TM is present. Besides acting as a benign tumor causing local bone destruction, the keratinaceous debris of a cholesteatoma is a good culture medium and may become a focus of infection for chronic OM. Complications include ossicular erosion with hearing loss, bone erosion into the inner ear with dizziness, or exposure of the dura, with consequent meningitis or a brain abscess. Cholesteatoma should be removed surgically after CT scan and hearing evaluation, and appropriate antibiotic therapy. A second-look procedure 6-9 months after primary surgery is often recommended to prevent further recurrence. Congenital cholesteatoma is an aggressive disease and needs to be taken care of to prevent permanent damage to the middle and inner ear. None of the following describe a cholestetoma and all refer to other conditions commonly associated with hyperlipidemia. Corneal arcus (B) is a greyish-white ring (or part of a ring) opacity occurring in the periphery of the cornea, in middle and old age. It is due to a lipid infiltration of the corneal stroma, with age the condition progresses to form a complete ring. That ring is separated from the limbus by a zone of clear cornea. Xanthelasma are localized deposits of fats that collect in the skin of the upper and lower eyelids (C). They can be skin-colored to yellowish and are associated about half the time with elevated blood lipids (fats), including cholesterol and triglycerides, which may be a sign of diabetes. Removal usually requires electric cautery or surgery. Cholesterol plaques are the culprits of heart disease. Plaques begin in artery walls (D) and grow over years. The growth of cholesterol plaques slowly blocks blood flow in the arteries. Worse, a cholesterol plaque can suddenly rupture. The sudden blood clot that forms over the rupture then causes a heart attack or stroke.

Which of the following drugs should be given empirically to an undifferentiated patient with altered mental status? Dextrose Flumazenil Naloxone Physostigmine

Correct Answer ( A ) Explanation: Hypoglycemia is a common cause of altered mental status (AMS) and should be rapidly treated in patients with altered mental status with intravenous dextrose. In undifferentiated patients with AMS, dextrose may rapidly reverse hypoglycemia and has minimal side effects. Hypoglycemia can present with a number of signs and symptoms ranging from irritability to focal neurological deficits to stupor and coma. Because of the varied presentations and lack of a hallmark finding on examination, empiric therapy has long been recommended. Additionally, the downside of administering dextrose to a patient who is normoglycemic is minimal. Flumazenil (B) is not empirically recommended because patients may have coingestions and reversing the benzodiazpene can result in withdrawal and seizures. Naloxone (C) is a reversal agent for opiates and should be given to those with opiate toxidromes. Physostigmine (D) is the reversal agent for anticholinergic toxidromes and should not be given empirically.

Which of the following antidotes is paired with the correct poisoning? Bupivacaine - Intralipid Hydrofluoric acid - Sodium bicarbonate Lorazepam - Fomepizole Metformin - Octreotide

Correct Answer ( A ) Explanation: Intralipid has emerged as a potential antidote to highly protein bound drugs such as local anesthetics, beta-blockers, calcium channel blockers and tricyclic antidepressants. There are several proposed mechanisms of action. The "lipid sink" theory proposes that highly fat-soluble drugs are soaked up by intralipid and removed from the site of toxicity decreasing the volume of distribution. The second mechanism is enhanced cardiac metabolism of fatty acids through a large supply of fat. The heart switches to glucose metabolism in stressful situations which may not suffice. The fatty substrate of intralipid supports better cardiac intracellular metabolism. Bupivacaine is an amide local anesthetic which has profound cardiac toxicity in higher doses and was one of the first agents for which intralipid was identified as an antidote in poisonings. Intralipid is administered at a dose of 1.5 ml/kg over 2-3 minutes followed by an infusion of 0.25 ml/kg/min. Hydrofluoric acid (B) poisoning usually occur through skin exposures. Calcium gluconate is the appropriate antidote administered topically when mixed into a gel or injected locally into the exposed area. It may also be injected intraarterially for severe exposures. Lorazepam (C) is a benzodiazepine and the appropriate antidote is flumazenil. In patients on chronic benzodiazepines, flumazenil may induce seizures and withdrawal so caution must be taken in its use. Fomepizole is the antidote for toxic alcohols (methanol and ethylene glycol). In large ingestions, metformin (D) may induce a lactic acidosis. There is no direct antidote for this medication and dialysis may be required. For the sulfonylurea class of oral hypoglycemics, an octreotide infusion is the appropriate antidote.

A 65-year-old man presents with ringing in his ears. He takes a number of medications. Which of the following medications is known to cause tinnitus at toxic levels? Acetylsalicylic acid Levetiracetam Meclizine Metoprolol

Correct Answer ( A ) Explanation: Salicylate toxicity can happen both in acute and chronic ingestions. In acute salicylism, the medullary respiratory center is stimulated leading to hyperventilation and respiratory alkalosis. It also causes a metabolic acidosis through an uncoupling of the oxidative phosphorylation pathway leading to an increased production of pyruvate that is ultimately converted to lactic acid. A life-threatening complication of acute salicylate toxicity is cerebral edema, the cause of which is not understood. Early in the course of toxicity patients experience nausea, vomiting, dehydration and hyperventilation. Over time, patients develop tinnitus and ultimately hearing loss. 0:00 Levetiracetam (B) does not cause tinnitus as part of its side effect profile. Patients may experience emotional lability, sedation, dizziness and even frequent colds. Meclizine (C) is an antihistamine medication used in the treatment of vertigo. Its most common side effect is sedation. However, in toxic quantities it may cause anticholinergic symptoms. Metoprolol (D) is a beta-blocker working both on the inotropy and chronotropy of the heart. In large doses, patients become hypotensive and bradycardic. In significant toxicity atrioventricular blocks develop. Patients may become hypoglycemic due to the activity of insulin transport stimulated by the beta-receptors.

What medication should be first-line treatment in moderate musculoskeletal back pain? Acetaminophen Cyclobenzaprine Diazepam Oxycodone

Correct Answer ( A ) Explanation: The first choice for patients with mild to moderate back pain should be acetaminophen or a nonsteroidal anti-inflammatory drug (NSAID) like ibuprofen or naprosyn. In patients with severe pain, opiate medications may be necessary. However, it is important to note that no studies have demonstrated the superiority of one pain medication (or class of medication) over another in the treatment of back pain. Patients with milder symptoms should initially be treated with acetaminophen or an NSAID. These medications are well tolerated in short courses with minor side effects. NSAIDs have not been shown to be superior to acetaminophen.

What level does the spinal cord terminate in adults? L1-L2 L2-L3 L3-L4 L4-L5

Correct Answer ( A ) Explanation: The level of the spinal cord varies by age as the spine grows. Knowing the level is essential knowledge prior to performing a lumbar puncture. In the fetus the spinal canal and spinal cord are equal in length. But, growth of the spinal cord does not keep pace with the growth of the spinal canal. In adults and older children the spinal cord (conus medullaris) terminates at the lower level of L1 spinal nerve or the body of L2. Therefore, the best approach for a lumbar puncture is between the L3-L4, L4-L5 or L5-S1 interspace. In infants, the cord terminates at L3 (B and C). Therefore, in infants a spinal needle should be placed in the L4-L5 (D) or L5-S1 interspace.

What is the most common cause of vertigo? Benign paroxysmal positional vertigo Labyrinthitis Meniere's disease Vertebrobasilar insufficiency

Correct Answer ( A ) Explanation: The majority of vertigo is caused by peripheral diseases and are not life-threatening. Benign paroxysmal positional vertigo (BPPV) is the most common of the peripheral disorders. Vertigo is defined as an illusion of motion. It is classically described as sensation of the room spinning. There are a number of causes of vertigo. The critical action in determining the cause is to separate peripheral vertigo from central vertigo. Peripheral vertigo can be caused by vestibular neuritis, labyrinthtits, Meniere's disease, acute otitis media, perilymphatic fistula, trauma to the labyrinth, acoustic neuroma and BPPV. Each of these etiologies have classic findings and symptoms that can aid in differentiating the causes. BPPV is typically caused by the presence of an otolith in the labyrinth system. The Dix-Hallpike maneuver can be used to support the diagnosis of BPPV. After performing a Dix-Hallpike, an Epley maneuver can be performed to expel the otolith from the labyrinth. Labyrinthitis (B) is typically preceded by an ear, nose and throat infection and is accompanied by hearing loss. Meniere's disease (C) typically has concurrent tinnitus with vertigo. Vertebrobasilar insufficiency (D) is a central cause of vertigo that may precede an ischemic CVA.

A 76-year-old man presents to the ED with three hours of diffuse abdominal pain. He has a past medical history of hypertension, CVA, and smoking. His current medications are metoprolol, warfarin, and lisinopril. His vital signs are pulse 72 beats per minute, blood pressure 78/40 mm Hg, respiratory rate is 20 breaths per minute, with an oxygen saturation of 98% on room air. On exam, the patient appears uncomfortable. His lungs are clear and his cardiac exam has a regular rhythm. His abdomen is mildly distended with diffuse tenderness to palpation without guarding or rebound. Which of the following is the most appropriate diagnostic test? Bedside ultrasound Portable chest X-ray Portable kidneys, ureters, and bladder X-ray Stat CT scan of the abdomen and pelvis

Correct Answer ( A ) Explanation: The patient is presenting with signs and symptoms of several possible abdominal disasters, most concerning of which is an expanding or ruptured abdominal aortic aneurysm (AAA). Bedside ultrasound is the test of choice in this case because it can be done immediately without need to move the patient out of the emergency department. Using bedside ultrasonography, appropriately trained emergency physicians can identify an AAA with a sensitivity of 100% and a specificity of between 98% and 100%. Although a portable chest x-ray (B) may identify free air under the diaphragm, this study takes longer to obtain, and the finding of free air is less important than knowing whether the patient has an AAA. Obtaining a portable kidneys, ureters, and bladder (KUB) X-ray (C) would not identify potential free fluid and only rarely can identify an AAA. Although a stat CT scan of the abdomen and pelvis (D) would also be useful to detect AAA, this patient is unstable and should not leave the emergency department for testing.

A 67-year-old woman presents with a gradual onset headache that has been present for several days. Examination is notable for tenderness over the left temple. Vital signs are normal. Which of the following is the most appropriate test to order? Erythrocyte sedimentation rate Lumbar puncture with opening pressure Non-contrast head computed tomography Outpatient magnetic resonance imaging

Correct Answer ( A ) Explanation: The patient's symptoms suggest temporal arteritis. Temporal arteritis is a form of giant cell arteritis affecting large and medium-sized vessels. Temporal arteritis occurs most commonly in patients over 50. The pain may occur anywhere on the head, but is usually localized to the temporal region. Up to half of patients may report a history of jaw claudication with chewing due to vascular insufficiency of the masseter and temporalis muscles. Patients may also develop polymyalgia rheumatica involving pain in large, proximal joints throughout the body. The erythrocyte sedimentation rate is markedly elevated in the majority of patients. Definitive diagnosis is made with a temporal artery biopsy. Treatment is with long-term high dose steroids and should not be delayed pending biopsy. Untreated temporal arteritis leads to visual loss in 50% of patients. Lumbar puncture with opening pressure (B) is an important part of the evaluation for patients with suspected meningitis or idiopathic intracranial hypertension. Non-contrast head computed tomography (C) is indicated in the evaluation of headaches suspicious for subarachnoid hemorrhage, mass, traumatic injury, or ventriculo-peritoneal shunt malfunction. Outpatient magnetic resonance imaging (D) is a consideration for non-acute headaches and the evaluation of possible structural lesions. Additionally, clinicians may use MRI for younger patients to avoid radiation exposure.

A 55-year-old man presents to the ED with blurry vision and difficulty speaking and swallowing. His symptoms began gradually. He also complains of a dry mouth, painful tongue, and sore throat. Which of the following is the most likely diagnosis? Botulism Diphtheria Eaton-Lambert Myasthenia gravis

Correct Answer ( A ) Explanation: There are 7 types of botulinum toxin (A through G) produced by C. botulinum, but only types A, B, E, and F cause illness in humans. Clinically, patients first exhibit cranial nerve dysfunction, with diplopia, dysphonia, dysphagia, and dysarthria being most prominent. Vertigo is also a common symptom. Subsequently, a descending symmetrical weakness occurs, involving the upper and lower extremities and the muscles of respiration. Decreased salivation (due to blockade of cholinergic fibers) causes a dry mouth, which may be so severe that patients complain of a painful tongue and sore throat. This constellation of symptoms should prompt the emergency physician to inquire about the ingestion of home-canned foods Diphtheria (B) is associated with descending paralysis. It is distinguished by the prolonged interval between pharyngitis and neurologic symptoms. Eaton-Lambert syndrome (C) is a rare autoimmune disease associated with descending paralysis but usually does not involve bulbar muscles. More than half of patients with Eaton-Lambert syndrome have an underlying malignancy, most commonly small cell lung cancer; it is therefore regarded as a paraneoplastic syndrome. Myasthenia gravis (D) is an autoimmune disease associated with antibodies that block acetylcholine receptors at the postsynaptic junction. The hallmark of myasthenia gravis is fatigability. Muscles become progressively weaker during periods of activity and improve after periods of rest. Muscles that control eye and eyelid movement, facial expressions, chewing, talking, and swallowing are especially susceptible. However, no autonomic symptoms are present.

76-year-old woman with a history of hypertension presents with left sided headache and jaw pain. She states that over the last four weeks, she has had progressively worsening headaches and the pain in her jaw makes it difficult to eat. She also complains of decreased vision in her left eye for the last 6 hours. Exam reveals tenderness over the left temple and jaw. Which of the following is the most appropriate next step in management? ESR, CRP and initiate corticosteroid therapy ESR, CRP and referral to a rheumatologist for further management Non-contrast head CT scan Treat her headache with NSAIDs and refer to an ophthalmologist

Correct Answer ( A ) Explanation: This patient has a presentation consistent with temporal arteritis or giant cell arteritis. Temporal arteritis is a large vessel disease characterized by granulomatous inflammation. It is most common in women aged 50-70 years. Onset of symptoms is usually gradual and can have systemic manifestations including fever, weight loss and anorexia. The classic symptoms are visual loss in one eye, temporal artery tenderness and jaw claudication. Headache is the most common presenting symptom. The diagnosis can be made based on clinical manifestations alone if classic symptoms are present. If the patient does not have classic symptoms, an elevated ESR (> 50 mm/hr) and CRP can aid in diagnosis. Ultimately, a temporal artery biopsy is needed to confirm the diagnosis. It is critical to initiate empiric treatment with corticosteroids. Any delay can result in permanent loss of vision. Steroid therapy should not be withheld while awaiting biopsy or follow-up (B) as permanent partial or complete vision loss in one or both eyes may occur in a short period of time. The likelihood of visual symptom resolution correlates with the time of corticosteroid therapy initiation. 50% of patients will have vision recovery if treated within 24 hours. This number falls to 6% in those treated after 24 hours. Control of symptoms and referral (D) is not indicated in older patients with a new headache and visual symptoms. A non-contrast head CT scan (C) is not useful in the workup as it cannot visualize abnormalities of the temporal artery.

What is the most common complaint seen in acute bronchitis? Chest pain Cough Fever Sputum

Correct Answer ( B ) Explanation: Bronchitis is defined as inflammation of the lower airways and is typically caused by viral pathogens. The most common presenting complaint is cough in these patients. In healthy adult patients, acute bronchitis is typically caused by a viral pathogen. As such, it is not amenable to antibiotic therapy. In patients with chronic bronchitis or chronic obstructive pulmonary disease (COPD), exacerbations of symptoms may be the result of bacterial pathogens causing acute bronchitis on top of their chronic bronchitis. Treatment with antibiotics should be considered in this subset of patients with bronchitis. Chest pain (A) often occurs as a result of coughing in patients with bronchitis. Fever (C) may or may not be present and does not indicate a higher likelihood of bacterial infection. Sputum (D) is present in about half of patients with acute bronchitis.

Which of the following should you inquire about in a two-year old male who presents with recent development of a large unilateral cervical lymph node? Contact with domesticated birds Exposure to cats Ingestion of raw shellfish Possible ingestion of lead-based paint

Correct Answer ( B ) Explanation: Cat scratch disease (CSD), also known as cat scratch fever or subacute regional lymphadenitis, develops postinoculation with Bartonella henselae, a gram-negative rod. CSD is one of the most common causes of chronic lymphadenopathy in children and adolescents. CSD is usually preceded by a scratch or bite from a cat (typically a kitten), which serves as the source of bacterial inoculation. The appearance of a papule at the inoculation site is the earliest symptom, followed by solitary or regional lymphadenopathy within 1-2 weeks. Single node involvement occurs in more than 50% of patients; 65% involve axillary or cervical lymph nodes. Spontaneous resolution within 2-4 months is the norm. Contact with domesticated birds (A) can result in Psittacosis, a respiratory infection with constitutional symptoms caused by the obligatory intracellular bacterium Chlamydia psittaci. Ingestion of lead-based paint (D) is the most common source of lead poisoning in children. Most homes built before 1978 were painted with lead-based paint. A small paint chip containing 50% lead can produce acute lead poisoning in a toddler. Since the effects of detrimental lead levels are often clinically silent, emphasis is placed on periodic screening in preschool children. Symptomatic lead poisoning has a wide-ranging clinical presentation but is not associated with unilateral lymphadenopathy. Eating raw shellfish (C) can lead to gastrointestinal disorders and, on occasion, neurotoxic manifestations, but it is not associated with isolated lymphadenopathy.

What is the most commonly seen symptom or sign in patients with acute aortic dissection? Aortic insufficiency murmur Chest pain Pulse deficit Syncope

Correct Answer ( B ) Explanation: Chest pain is the most common symptom seen in patients with acute aortic dissection. Aortic dissection is an uncommon presentation but it represents a difficult and life-threatening diagnosis. Difficulty in diagnosing the disease stems from the myriad of presentations and manifestations the disease can assume. Approximately 73% of patients with acute aortic dissection will present complaining of chest pain. This symptom is more common in those patients with ascending dissections whereas back pain is more common in those with descending thoracic dissections. Some complaint of pain is seen in up to 96% of patients. The pain is classically described as ripping or tearing but only about 50% of patients will describe it in this way. Aortic insufficiency murmur (A) results from an ascending dissection that compromises the aortic valve but is only seen in about 32% of patients. Pulse deficit (C) is even less common (15%). Syncope (D) is seen in approximately 9% of aortic dissections.

Parents bring their one-month-old infant in for evaluation because of excessive crying and fussiness. Which of the following is associated with a pathologic diagnosis? Continued crying after the application of tetracaine to the eye Finger swelling Mustard colored stool Soothing when put in parent's arms

Correct Answer ( B ) Explanation: Crying is the infant's response to physiologic needs like hunger, cold or discomfort. However, excessive crying may also suggest a serious illness and it is important for the clinician to differentiate these infants from others. Maternal history (e.g. medications, drugs) and infant history (birth history, stool habits, feeding) may help to guide the clinician. Physical examination findings revealed the diagnosis in less than half in a case series of crying infants. A swollen digit may be a clue to a hair tourniquet constricting the finger and causing pain. It may not be obvious to the parents until the finger becomes grossly swollen and discolored. This can result in local ischemia, necrosis, and autoamputation. In any infant with inconsolable crying, this syndrome must be excluded.

An 87-year-old woman presents to the ED after her caregiver witnessed the patient having difficulty swallowing over the past 2 days. The patient is having difficulty with both solids and liquids. She requires multiple swallowing attempts and occasionally has a mild choking episode. She has no other complaints. Your exam is unremarkable. Which of the following is the most likely cause of her condition? Achalasia Cerebrovascular accident Esophageal neoplasm Foreign body

Correct Answer ( B ) Explanation: Dysphagia can be divided into two categories: transfer and transport. Transfer dysphagia occurs early in swallowing and is often described by the patient as difficulty with initiation of swallowing. Transport dysphagia occurs due to impaired movement of the bolus down the esophagus and through the lower sphincter. This patient is experiencing a transfer dysphagia. This condition is most commonly due to neuromuscular disorders that result in misdirection of the food bolus and requires repeated swallowing attempts. A cerebrovascular accident (stroke) that causes muscle weakness of the oropharyngeal muscles is frequently the underlying cause. Achalasia (A) is the most common motility disorder producing dysphagia. It is typically seen in patients between 20 and 40 years of age and is associated with esophageal spasm, chest pain, and odynophagia. Esophageal neoplasm (C) usually leads to dysphagia over a period of months and progresses from symptoms with solids to liquids. It is also associated with weight loss and bleeding. Foreign bodies (D) such as a food bolus can lead to dysphagia, but patients are typically unable to tolerate secretions and are often observed drooling. These patients do not have difficulty in initiating swallowing.

A 75-year-old man on chemotherapy for metastatic prostate cancer presents with a fever. His white blood cell count is 1,200 cells/l with 22% neutrophils, 44% lymphocytes, 8% monocytes, and 9% bands. Which of the following mandates antibiotic treatment? Oral temperature of 100.4°F measured once Oral temperature of 101°F measured once Rectal temperature of 100°F for one hour Rectal temperature of 99.6°F measured for one hour

Correct Answer ( B ) Explanation: In patients on chemotherapy, the development of neutropenia is a known complication due to suppression of the bone marrows production of all cell lines from cytotoxic chemotherapy. When evaluating these patients, it is critical to calculate the absolute neutrophil count (total white blood cell count x % neutrophils). Neutropenia is defined as an absolute neutrophil count of < 500 cells/L or an ANC that is expected to decrease to this level within 48 hours. If the patient is neutropenic, guidelines have been established to define what constitutes a fever. The Infectious Disease Society of America defines neutropenic fever as a single oral temperature measurement of101F (38.3C). Additionally, two oral temperature measurements measured at least one hour apart of100.4F (38C) is also considered a fever. Oral temperature of 100.4F measured once (A), rectal temperature of 100F for one hour (C), and rectal temperature of 99.6F measured for one hour (D) does not constitute neutropenic fever. It is important to recognize that the measurement of a rectal temperature and the performance of a digital rectal examination are not recommended in neutropenic patients. In theory, this maneuver may increase bacterial translocation through mucosa that is inflamed as a result of chemotherapy.

A 18-year-old man presents with a depressed mental status. Which of the following could be empirically given? Activated charcoal Dextrose and thiamine Flumazenil Syrup of ipecac

Correct Answer ( B ) Explanation: In the undifferentiated patient with depressed mental status, empiric delivery of dextrose and thiamine should be considered. Hypoglycemia can manifest with varied presentations. Patients may exhibit lethargy, somnolence, agitation, confusion, altered mental status, seizures, focal neurologic deficits or coma. In all patients with neurologic changes, a rapid finger stick should be obtained. However, in the absence of a finger stick, it is reasonable to empirically treat with dextrose as hypoglycemia is life threatening. 1-2 grams of dextrose/kg should be given intravenously. In patients in whom intravenous access cannot be established, glucagon can be given intramuscularly. This will increase circulating glucose levels by mobilizing glycogen stores. However, it will not work in those patients with limited glycogen stores (chronically ill, malnourished etc.). Thiamine should be given with dextrose to prevent the development of Wernicke's encephalopathy although the traditional "thiamine before dextrose" has little basis in the literature. Activated charcoal (A) should not be given to patients with depressed mental status if their airway is not protected as aspiration of activated charcoal is extremely hazardous. Flumazenil (C) is a benzodiazepine antagonist. It should not be given empirically as it removes the ability to control seizures in patients taking first line medications. Syrup of ipecac (D) induces vomiting and can lead to aspiration in a patient not protecting their airway.

A 50-year-old man presents with episodic severe vertigo lasting hours, with associated symptoms of unilateral tinnitus, fluctuating low-frequency hearing loss, and aural fullness. Which of the following is the most likely diagnosis? Benign paroxysmal positional vertigo Meniere's disease Perilymph fistula Vestibular neuronitis

Correct Answer ( B ) Explanation: Meniere's disease is characterized by episodic severe vertigo lasting hours, with associated symptoms of unilateral tinnitus, fluctuating low-frequency hearing loss, and aural fullness. Typical onset is in the fifth decade of life. The cause is uncertain but is speculated to result from allergic, infectious, or autoimmune injury. The histopathologic finding includes endolymphatic hydrops, which is thought to be caused by either overproduction or underresorption of endolymph in the inner ear. Meniere's disease is a clinical diagnosis mostly based on history. Testing may be obtained to support the diagnosis and rule out other disorders. Audiometry often demonstrates a low-frequency sensorineural hearing loss. An FTA-ABS test may be obtained to rule out syphilis. Electronystagmography (ENG) may demonstrate a unilateral peripheral vestibular weakness on caloric testing. When the diagnosis is uncertain, a brain MRI with contrast is obtained to evaluate for a retrocochlear lesion. The differential diagnosis of Meniere's disease includes acute labyrinthitis, neurosyphilis, labyrinthine fistula, autoimmune inner ear disease, vestibular neuronitis, and migraine-associated vertigo. The most common cause of peripheral vestibular vertigo in adults is benign paroxysmal positional vertigo (A). BPPV occurs in all age groups but more often between ages 50 and 70 but is not associated with hearing loss and made worse with movement. In a perilymph fistula (C) rapid changes in air pressure (barotrauma), otologic surgery, violent nose blowing or sneezing, head trauma, or chronic ear disease may cause leakage of perilymph fluid from the inner ear into the middle ear and result in episodes of vertigo. Associated signs and symptoms are variable but can include a sudden pop in the ear followed by hearing loss, vertigo, and sometimes tinnitus. Acute vertigo associated with nausea and vomiting (but without neurologic or audiologic symptoms) that originates in the vestibular nerve is known as vestibular neuronitis (D). Vestibular neuronitis can occur spontaneously or can follow viral illness.

Nitrate therapy works by which of the following mechanisms? Reducing afterload Reducing both preload and afterload Reducing cardiac contractility Reducing preload

Correct Answer ( B ) Explanation: Nitrates work by reducing both preload and afterload. Nitroglycerine was the first treatment for angina pectoris and dates back to the 1800s. It still remains first-line drug therapy for many patients. Nitrates dilate veins and coronary arteries and to a lesser extent systemic arteries by relaxing vascular smooth muscle. Thus, nitrates reduce preload by increasing venous capacitance and improve coronary blood flow by coronary vasodilatation. Decreased preload lowers left ventricular end-diastolic pressure, thereby decreasing wall stress, resulting in a decrease in myocardial oxygen demand. At higher doses the afterload effects cause a drop in systemic blood pressure further decreasing wall stress and oxygen demand. Nitrates do not have a direct effect on cardiac chronotropy or inotropy. The primary adverse effects induced by nitrate therapy include hypotension, headache, and tachycardia. Nitrates should be avoided in patients with one or more of the following: systolic blood pressure less than 90 mm Hg, heart rate less than 50/min, or heart rate greater than 100/min. It should also be avoided in known or suspected right ventricular infarction, in patients who have taken a phosphodiesterase inhibitor for erectile dysfunction within the last 24 hours, in patients with hypertrophic cardiomyopathy or severe aortic stenosis.

In a patient with suspected HIV infection and shortness of breath, which of the following findings is most suggestive of Pneumocystis jiroveci pneumonia? Bradycardia despite relative volume depletion Elevated serum lactate dehydrogenase Hyponatremia Unilateral lobar consolidation on chest radiograph

Correct Answer ( B ) Explanation: Pneumocystis pneumonia is one of the most common opportunistic infections in patients with HIV. Patients typically present with an insidious nonproductive cough, dyspnea, unexplained fever for longer than two weeks, chest pain, and fatigue. The greater the elevation of lactate dehydrogenase (LDH), the worse the prognosis. Relative bradycardia (A) is commonly associated with typhoid fever (Salmonella typhi), Legionnaire's disease (Legionella pneumophila), infectious mononucleosis, and pneumonia caused by Chlamydia species. The syndrome of inappropriate antidiuretic hormone secretion (SIADH) is one of several causes of hyponatremia (C). SIADH may be seen in patients with certain pulmonary infections, including Legionnaire's disease and tuberculosis, though the mechanism is poorly understood. SIADH has not been reported with Pneumocystis jiroveci pneumonia. The chest radiograph (D) in Pneumocystis jiroveci pneumonia commonly shows a diffuse interstitial infiltrate but may also be normal in appearance or reveal asymmetrical nodules, cavitation, or bullae.

Which of the symptoms are most suggestive of Ramsay Hunt syndrome? Bilateral petechiae of the palate Ear pain, facial paralysis, hearing loss Lymphangitis with thrombus formation in the internal jugular vein Photophobia and unilateral thoracic vesicular rash

Correct Answer ( B ) Explanation: Ramsey Hunt syndrome is caused by herpes zoster and is characterized by unilateral facial paralysis, a herpetiform vesicular eruption, and vestibulocochlear dysfunction (hearing loss). The vesicular eruption may occur on the pinna, external auditory canal, tympanic membrane, soft palate, oral cavity, face, neck, and as far down as the shoulder. Although it shares some clinical features with Bell's palsy, Ramsay Hunt syndrome is often associated with severe pain. Treatment includes steroids and antiviral therapy. Bilateral petechiae of the palate (A) are associated with infectious mononucleosis and Streptococcal pharyngitis. Lymphangitis with thrombus formation in the internal jugular vein (C) is characteristic of Lemierre's syndrome. This occurs after infection of the oropharynx. The causative pathogen is Fusobacterium necrophorum, a gram-negative anaerobic bacillus. Photophobia and unilateral thoracic vesicular rash (D) are characteristic of herpes zoster infection. Ramsay Hunt syndrome is also caused by the herpes zoster virus but is associated with infection involving cranial nerves VII and VIII.

40-year-old woman with a history of hypertension, diet-controlled diabetes, and CHF presents to the ED via EMS after a suspected syncopal episode. Witnesses saw the patient slump to the ground, noticed some jerking in her arms, and then observed the patient wake up within 10 seconds. In the ED, the patient is currently asymptomatic. Vital signs are 162/94, HR 85, RR 14, and T 36.8°C. Her ECG shows a sinus rhythm with a rate of 88. Her complete blood count and basic metabolic panel are within normal limits except for a serum glucose of 230. The urine pregnancy test is negative. Which of the following is the most appropriate disposition for this patient? Admission for further risk stratification based on current blood pressure Admission for further risk stratification based on past medical history Discharge after reduction of blood pressure and correction of blood glucose with close outpatient follow-up Discharge with neurology follow-up because this event most likely represents a seizure

Correct Answer ( B ) Explanation: Syncope is a common presentation in the ED. Several decisions rules exist to assist in clinical decision making on which patients are safe to be discharged and which require further inpatient workup. Despite heterogeneity of the rules, most encourage admission in patients with high-risk features, which can include an abnormal ECG, a history of structural heart disease or heart failure, persistent hypotension, shortness of breath, hematocrit less than 30%, family history of sudden cardiac death, or older age/presence of multiple comorbidities. Given this patient's history of CHF, she should be admitted for further evaluation. It is often difficult to discern between syncope and seizure. Confusion most commonly occurs after a seizure rather than after syncope. Clonic jerking can occur with syncope that is often confused with a seizure. The patient's clinical presentation is more consistent with syncope than with seizure (D). A mild elevation in blood pressure (A) is not considered a high-risk feature of syncope. Emergent lowering of blood pressure (C) is not indicated in this patient. However, given her history of CHF, she is at increased risk for an adverse event and should be admitted for further evaluation. Her serum blood glucose can be corrected as an inpatient.

Which of the following is the main nerve root that supplies innervation to the diaphragm? C2 C4 C6 C8

Correct Answer ( B ) Explanation: The C4 nerve root is the main root that innervates the diaphragm. The phrenic nerve consists of roots C3, C4, and C5, but the C4 nerve root plays the most significant role in the motor function of the diaphragm. Therefore, injury at or above C4 is usually life threatening. Complete transection at or above C4 is generally fatal because there is a loss of diaphragmatic and respiratory muscle motor function. A common mnemonic is "c345 keeps the phrenic alive" (innervation of phrenic nerve), and "c345 keeps the diaphragm alive" (innervation of the diaphragm). The C2 nerve root (A) provides innervation for the anterior and lateralis rectus capitis, omohyoid, sternohyoid, sternothyroid, longus capitis, and longus colli muscles. The C6 nerve root (C) is part of the brachial plexus and innervates multiple muscles involving the back, chest, and arms. The C8 nerve root (D) also forms the brachial plexus and innervates muscles of the chest, back, arms, and hands. However, only the C3, C4, and C5 nerve roots provide innervation to the diaphragm.

Which of the following fish poisonings is associated with a peppery taste? Aeromonas Ciguatera Scombroid Vibrio

Correct Answer ( C ) Explanation: Scombroid fish poisoning results from ingestion of particular fish (tuna, mackerel, bonito, herring, anchovy, sardine, and mahi mahi, among others) that contain heat-stable toxins produced by bacterial action on histidine present in the dark meat of the fish. While eating the fish, the patient may note a metallic, bitter, or peppery taste. However, fish with low levels of histamine may have no change in taste but can still cause symptoms. Symptoms start abruptly and consist of facial flushing, diarrhea, severe throbbing headache, palpitations, and abdominal cramps Aeromonas (A) and Vibrio (D) species are ubiquitous in fresh and brackish water and produce acute gastroenteritis. Neither bacteria alters the flavor of the fish. Ciguatera (B) poisoning results from the ingestion of ciguatoxin which is odorless and tasteless.

A 92-year-old man presents after falling down stairs and hitting his head. He opens his eyes to voice and intermittently makes sounds. He does not have any purposeful movement and periodically extends his arms and legs. What is his Glasgow Coma Scale score? 6 7 8 9

Correct Answer ( B ) Explanation: The patient's total Glasgow Coma Scale is 7. Assessment of the patient's eye opening, verbal response and motor response make up the score. The chart below indicates how the GCS is calculated. This patient receives the following scores: Eye movement 3, Verbal response 2, Motor Response 2. Eye opening and verbal responses are straightforward in the scoring system. For the motor calculation, knowledge of the difference between decerebrate and decorticate posturing is essential. Decerebrate posturing receives a lower score because it is associated with a worsened neurologic outcome, often associated with uncal herniation. In decerebrate posturing, the arms become extended at the elbow and adducted. The wrists and fingers are flexed and the entire arm is internally rotated. Similarly, the legs become extended and internally rotated with plantar flexion of the feet and toes. Decorticate posturing involves flexion of the upper extremity with extension of the lower extremity. The leg also internally rotates with plantar flexion of the feet and toes as in decerebrate posturing. Decorticate posturing is associated with injuries above the midbrain.

A 55-year-old man, smoker, presents to the ED with hemoptysis and dyspnea for four weeks. His vital signs are T 37°C, BP 146/76 mm Hg, HR 85 bpm, RR 20 per minute, and oxygen saturation 96% on RA. His lung exam reveals distant breath sounds on the left side. His chest X-ray is shown above. What is the most likely cause of his hemoptysis? Bronchitis Lung cancer Pneumonia Pulmonary embolism

Correct Answer ( B ) Explanation: This patient has a lung mass that has eroded into a pulmonary vessel. The pulmonary blood supply is under low pressure, so bleeding can present as blood-tinged sputum or gross hemoptysis. Massive hemoptysis is caused by hemorrhage from a bronchial vessel in the majority of cases because bronchial vessels are under systemic blood pressure. Some 80%-90% of patients with cancer as the cause of hemoptysis will have abnormalities on the chest X-ray. Those with normal chest x-rays with a high suspicion for an underlying neoplasm should undergo a chest CT scan and have an outpatient fiber-optic bronchoscopy. Although bronchitis (A) is the most common cause of hemoptysis (responsible for 15%-30% of cases), patients present with cough as the dominant symptom and have abnormal lung exams and normal chest x-rays. The cough may be productive of sputum. The diagnosis of pneumonia (C) requires focal findings on physical exam or infiltrates on radiographic imaging and is typically accompanied by a fever. Patients with lung cancer are at increased risk for pulmonary embolism (D). This patient's Wells score is one (one point for hemoptysis), which makes the likelihood of PE 1.3% in an ED population. Given the lung mass seen on the chest x-ray, lung cancer is more likely than PE.

Which of the following patients requires radiologic imaging for her back pain? A 26-year-old woman with unilateral costovertebral angle tenderness, fever, and dysuria A 30-year-old woman with midline lumbar pain and tenderness, fever, and difficulty urinating A 35-year-old woman with bilateral lumbar level back pain after lifting furniture that is worse with movement and alleviated with rest A 40-year-old woman with lumbar-level pain and tenderness that radiates to her left anterior thigh, with a normal neurologic exam

Correct Answer ( B ) Explanation: This patient has symptoms suspicious for a spinal epidural abscess. The classic triad is severe back pain, fever, and neurological deficits. In her case, urinary retention is her deficit, which by itself is worrisome for cauda equina syndrome. The diagnostic test of choice is MRI. Epidural abscess may require evacuation by a spine surgeon. Patients are often on IV antibiotics for many weeks. However, to avoid sterilization of the abscess and facilitate isolation of the causative organism, it is recommended that antibiotic therapy be withheld in the emergency department for suspected (or even radiographically confirmed) cases. Patients at risk for spinal epidural abscess include intravenous drug users, immunocompromised patients, alcohol abusers, diabetics, patients with malignancy, and those with recent spine procedures. Pyelonephritis (A) is an infection of the renal parenchyma and upper collecting system. In an otherwise healthy, young patient, treatment can be performed on an outpatient basis with current guidelines recommending 14 days of antibiotics. Radiographic imaging is not needed unless there is suspicion of an infected kidney stone. Nonspecific back pain (C) in the absence of "red flags" (e.g., fever, neurologic deficit, IV drug use, trauma) does not require radiographic imaging and can be managed symptomatically with resumption of usual daily activities as soon as possible. Bed rest has been shown to prolong the duration of pain, as have back exercises. Most nonspecific back pain resolves in 4-6 weeks. Radiculopathy (D) or nerve impingement syndrome may be caused by a disk herniation or other irritation of the nerve root. Given the location of symptoms, compression of the L4-L5 nerve root should be suspected but without worrisome features; imaging in the emergency department is not indicated. If her symptoms do not resolve in 4-6 weeks, an MRI of the lumbosacral spine may be warranted.

A 34-year-old man presents to the emergency department with facial flushing, nausea, vomiting, and palpitations after eating tuna just one hour prior to presentation. His vital signs are T 36.7°C, HR 120 bpm, BP 130/70 mm Hg, and RR 18. On physical exam he appears diaphoretic and anxious. He has normal bowel sounds and no abdominal tenderness on exam. The rest of his exam is unremarkable. What is the most likely etiology of this patient's symptoms? Ciguatera Histamine fish poisoning Staphylococcus Tetrodotoxin Vibrio parahaemolyticus

Correct Answer ( B ) Explanation: This patient's presentation is most consistent with histamine fish poisoning. Histamine fish poisoning is a food-related illness that occurs because of consumption of improperly stored fish. Bacteria on the surface of the fish decarboxylate histidine, a ubiquitous amino acid, forms a histamine. Commonly implicated fish include the following: tuna, mackerel, and mahi-mahi. When ingested, the histamine is absorbed and causes symptoms of flushing, headache, vomiting, abdominal pain, palpitations, and anxiety. Focal neurological findings are uncommon. The onset of symptoms is rapid, generally within the hour. Many species of fish can cause ciguatera poisoning (A), such as sea bass, grouper, and red snapper. Ciguatoxin increases neuronal sodium channel permeability leading to symptoms such as diaphoresis, headaches, abdominal pain, and cramps. Distinct neurologic symptoms, including the sensation of loose or painful teeth, parasthesias of the face and tongue, metallic taste, and reversal of temperature discrimination may also be reported. Staphylococcus (C) food poisoning is due to ingestion of a preformed toxin that leads to the onset of gastrointestinal symptoms (nausea, vomiting, diarrhea, and abdominal cramping) within 6 hours. Histamine-mediated symptoms do not occur. Tetrodoxin (D) is found in puffer fish and certain species of octopus. Marine bacteria that accumulate in fish produce the toxin, which inhibits sodium channels. Symptoms can occur within minutes, including headache, parasthesias, and facial nerve palsies. These may be followed by gastrointestinal disturbances and ultimately muscle weakness that progresses to an ascending paralysis. Death can occur from respiratory muscle paralysis. Care is supportive, and prolonged intubation may be required. Vibrio paraheamolyticus (E) infections are usually contracted by eating shellfish. Patients can present with nausea, vomiting, abdominal pain, diarrhea that may be bloody, fevers, and myalgias.

A 3-year-old boy presents with severe vomiting and diarrhea. The exam reveals sunken eyes, skin tenting, and a capillary refill of three seconds. At his last well visit he weighed 15 kg, but weighs just 13 kg today. Using the Holliday-Segar method, what is his maintenance fluid rate per hour? 260.0 ml/hr 39.0 ml/hr 47.9 ml/hr 83.3 ml/hr

Correct Answer ( C ) Explanation: 47.9 ml/hr is calculated utilizing the Holliday-Segar method of 100 ml/kg/day for the first 10 kg, 50 ml/kg/day for the next 10 kg, and 20 ml/kg/day for each additional kilogram. The total is then divided by 24 hours. (100 ml/kg/day x 10 kg) + (50 ml/kg/day x 3kg) = 1150 ml/day/24hr = 47.9ml/hr This method is sometimes shortened to the 4-2-1 method of 4 ml/kg/hr for the first 10 kg, 2 ml/kg/hr for the next 10 kg, and 1 ml/kg/hr for each additional kilogram. (4 ml/hr x 10 kg) + (2 ml/hr x 3 kg) = 46 ml/hr If isonatremic, sodium and potassium maintenance would need to be calculated, and added into this maintenance fluid for a proper order to be written. 39.0 ml/hr (B) is similar to the Parkland formula of fluid replacement that is utilized with burn patients. This formula for children is 3 ml x weight (kg) x % body surface area burned + maintenance. For adults, the formula is 4 ml x weight (kg) x % body surface area + maintenance. The fluid is replaced in increments with the one half given over the first 8 hours, and the remainder given over the next 16 hours. Lactated ringers is the fluid of choice for burns. 83.3 ml/hr (D) is this patient's fluid deficit. This is calculated as Fluid deficit (L) = pre-illness weight (kg) - illness weight (kg). In this case, 15kg - 13kg = 2L. The 2 L fluid deficit is divided by 24 hours to yield 83.3 ml/hr. If the pre-illness weight is unavailable, clinical observations may be utilized to determine the percent dehydration and calculated as percent (%) dehydrated x weight (kg) = Liters down. 260ml/hr (A) is the emergency bolus delivered and is done as quickly as possible. Boluses are delivered at the rate of 20 ml/kg unless a cardiac condition is present or suspected.

A 3-year-old girl presents to the ED with her mom and grandmother for evaluation of vaginal spotting. The child has no other complaints and specifically denies sexual abuse or trauma when questioned alone. After watching a news special on sexual abuse, mom is concerned that her daughter may have been sexually assaulted, despite no specific concern. The child is acting appropriately and there are no external lesions or signs of trauma. Internal pelvic exam is difficult due to the patient's age. Which of the following statements is true? Obtain a pelvic x-ray prior to attempting a physical exam She is the victim of abuse; do no further evaluation without a trained nurse present and contacting authorities She may have a vaginal foreign body; consider a nasal speculum to attempt visualization and removal Vaginitis is an unusual diagnosis in this age group

Correct Answer ( C ) Explanation: Children often fear parental disapproval of a vaginal foreign body placement. This often leads to a delay in diagnosis until secondary signs (such as vaginal bleeding, foul smelling discharge, or purulent drainage) are noted by parents. The physical exam is often quite difficult owing to patient anxiety, small anatomic size, and parental concerns about sexual or physical abuse. However, a thorough vaginal exam is indicated in patients with vaginal bleeding or discharge. Use of a nasal speculum, procedural sedation, or trained assistant (such as a child life specialist) may facilitate this exam. Although plain films (A) may identify certain radiopaque objects, they are often of limited diagnostic value. A negative film does not disprove the presence of a radiolucent vaginal foreign body. Sexual and physical abuse (B) is always an important consideration when evaluating children with genitourinary or rectal complaints. When in doubt, ethical and legal requirements obligate the notification of appropriate child protection services. However, in this case, there is no evidence of abuse. Foreign body or vaginitis is a much more likely diagnosis. Vulvovaginitis (D) is a very common diagnosis in this age range owing to a lack of estrogen, developing toilet hygiene, and frequent use of baths, perfumed soaps, and bubbles.

A 67-year old woman presents to the ED complaining of intermittent episodes of crushing substernal chest pain radiating to the left jaw and left arm. Blood pressure is 85/50 mm Hg, heart rate is 89 beats per minute, and the respiratory rate is 16 breaths per minute. Electrocardiogram shows ST segment elevation in leads II, III, and aVF with ST segment depression in lead aVL. The patient is started on 2 liters oxygen by nasal cannula, and peripheral intravenous access is obtained. Which of the following is the most appropriate next step in management? Dobutamine Metoprolol Normal saline bolus Sublingual nitroglycerin

Correct Answer ( C ) Explanation: ECG evidence of an inferior wall infarction (elevation in leads II, III, and aVF) in the presence of elevated jugular venous pressure or hypotension is concerning for a right ventricular infarction. Intravenous fluids are the initial therapy of choice and will increase preload while enhancing perfusion. It is critical to avoid preload reduction with venodilators such as nitroglycerin (D) in this setting of an inferior wall infarction. Dobutamine (A) is an inotropic agent reserved for cardiogenic shock and should be considered only in the event that the patient's blood pressure does not respond to fluid administration. Metoprolol (B) should be used cautiously in the presence of an inferior wall infarction because it can worsen conduction abnormalities through an ischemic AV node. A PR interval of 220 ms or greater increases the likelihood of AV node ischemia and is considered a relative contraindication to beta-blocker use; presence of any degree of AV block is considered an absolute contraindication.

A 21-year-old nurse presents after a needle stick from a needle that was uncapped and sticking out of the sharps container. Which of the following is most accurate in terms of HIV prophylaxis? Antiretroviral medications are taken for two weeks Post-exposure prophylaxis decreases rates of seroconversion by 25% Post-exposure prophylaxis started within two hours improves transmission prevention The window for initiation of treatment is 96 hours

Correct Answer ( C ) Explanation: HIV prophylaxis is indicated for a needle stick injury where there is any concern about the possibility of transmission of HIV. The risk of acquiring HIV from an infected needle stick is on average 0.3%. Other factors that play into the likelihood of transmission include the stage of illness of the source patient, depth of injury and whether blood was visible on the device. The most effective treatment is when the first dose of medication is administered within two hours of exposure. Exposed persons are started on antiretroviral medications that prevent HIV from replicating to a degree that is capable of causing seroconversion. The CDC recommends a two or three drug regimen based on the risk of exposure. Some states have their own guidelines established about particular medication. Antiretroviral medication is started immediately and continued for 28 days, not for two weeks (A). This is based on animal data demonstrating the effectiveness of treatment for this duration of time. Rates of effectiveness diminish if the entire course of medication is not completed. Post-exposure prophylaxis decreases rates of seroconversion by more than 50% (B). The one case-control series of occupational exposures showed decreased rates of transmission by 81% with the administration of only one drug (zidovudine). The window for initiation of treatment according to the CDC is 72 hours, not 96 hours (D). Animal data suggests that the window is shorter and probably closer to 36 hours.

A 50-year-old man, nonsmoker, presents to the ED with a 2-day history of cough now associated with frank hemoptysis. He denies any constitutional symptoms. Vital signs are BP 125/70, HR 80, RR 16, and pulse oximetry is 98% on room air. On exam, his lung fields are clear; the remainder of the exam is unremarkable. A chest radiograph is performed, which is normal. Which of the following is the most appropriate next step in management? Admit to the ICU Immediate bronchoscopy Obtain a CT scan of the chest Treat for bronchitis and discharge

Correct Answer ( C ) Explanation: Hemoptysis is classified as mild, moderate, or severe based on volume. In reality, it is very difficult for patients to quantify the amount of hemoptysis, so more relevant terminology in the ED should include speckled or blood-streaked sputum, gross hemoptysis (clotted or unclotted blood), and massive hemoptysis interfering with respiration. In the patient with nonmassive hemoptysis and stable vital signs, it is important to first rule out an alternative source of bleeding such as epistaxis or upper GI. A chest radiograph should then be obtained; if negative and the patient is still experiencing hemoptysis, a CT scan of the chest should be obtained. Consultation with a pulmonologist for possible bronchoscopy may also be needed. The patient is hemodynamically stable with a normal chest radiograph, so he does not require ICU admission (A). Patients with massive hemoptysis require ICU admission. The decision to perform a bronchoscopy (B) in this patient will be left up to the pulmonologist. Given the overall clinical picture, urgent bronchoscopy is not required in this case. With massive hemoptysis, an emergent bronchoscopy is indicated. Bronchitis (D) typically presents with the abrupt onset of cough with blood-streaked purulent sputum. The patient in the clinical scenario has persistent frank hemoptysis, which mandates further investigation. In a patient who does not smoke, is under the age of 40, and has a normal chest radiograph and scant hemoptysis, treatment for bronchitis can be initiated with outpatient follow-up.

A 35-year-old woman presents to the ED complaining of episodes of palpitations with lightheadedness over the last three days. Blood pressure is 120/70 mm Hg, heart rate 115 beats per minute, and respiratory rate is 18 breaths per minute. On cardiac examination, you note a midsystolic click. Which of the following is the most likely diagnosis? Aortic valve stenosis Hypertrophic cardiomyopathy Mitral valve prolapse Tricuspid valve stenosis

Correct Answer ( C ) Explanation: Mitral valve prolapse is due to dysfunction of the papillary muscles or chordae tendineae, leading to brief periods of regurgitation into the left atrium. This results in a normal S1 heart sound, followed by a characteristic midsystolic click and brief crescendo-decrescendo murmur. Murmurs due to mitral valve prolapse are made louder by Valsalva maneuvers. Squatting can lead to a delayed click, and can cause decreased intensity, but has also been known to increase the murmur due to increased afterload. The murmur of hypertrophic cardiomyopathy (B) also is made louder by Valsalva maneuvers but is not associated with a midsystolic click. The diagnosis is also much less common in women and is associated with more worrisome clinical consequences, including exertional syncope and sudden cardiac death. Aortic valve stenosis (A) is associated with a coarse, high-pitched crescendo-decrescendo systolic murmur radiating to the carotids. If not obscured by mitral stenosis sounds, a tricuspid opening snap may be heard in tricuspid valve stenosis (D). In addition, a diastolic murmur is audible along the left sternal border or at the xiphoid, which increases with inspiration.

A full-term neonate is brought to the ED for constant crying for the last three hours. In the ED, he sleeps quietly in his mother's arms. He cries when you examine him but is immediately consoled when he is swaddled and held. His exam is normal. What is the next most appropriate step in his management? CT scan of the head to rule out intracranial hemorrhage Lumbar puncture to rule out serious bacterial infection Reassurance Skeletal survey for abuse

Correct Answer ( C ) Explanation: Neonates cry in varying patterns throughout the day. Although crying is typically a sign of hunger or normal uncomfortable states (wet diaper, gas), it may also signal pain or underlying disease. An easily consoled infant without a source of crying after a thorough history and physical exam can be discharged with parental reassurance. A thorough exam for crying includes completely exposing the infant; palpating the fontanelles; fluorescein staining of the eyes for corneal abrasions; examining the fingers, toes, and penis for hair tourniquets; and checking for hernias. It is also important to look for retinal hemorrhages and palpate the extremities for accidental or deliberate trauma. Intracranial hemorrhage (A) may be caused by neonatal alloimmune thrombocytopenia, birth trauma, vitamin K deficiency, or acute injury. Symptoms of intracranial hemorrhage are irritability, inconsolable crying, poor feeding, and lethargy. Signs include a bulging fontanelle, retinal hemorrhages, and bruising. Serious bacterial infection (SBI) (B) may be present in any neonate but will usually present with more than just crying. The patient may have feeding difficulties, jaundice, hypo- or hyperthermia, tachypnea, or respiratory distress. Neonates with suspected sepsis must undergo a full septic workup with blood cultures, urine cultures, and CSF evaluation. They should be started on empiric antibiotics until cultures are negative. Neonatal child abuse (D) may be difficult to diagnose. Signs to look for include those correlating with underlying intracranial hemorrhage (see explanation for choice A), bruising, or deformity. Appropriate authorities must be contacted if child abuse is suspected, even if it is not confirmed.

Which of the following can be used to distinguish a seizure from a syncopal episode? Loss of consciousness Loss of urinary continence Post-ictal period Tongue biting

Correct Answer ( C ) Explanation: Syncope and seizure are often confused and difficult to differentiate. However, patients with syncope should not experience a post-ictal period or prolonged state of confusion. Syncope is defined as a sudden, transient loss of consciousness along with a loss of postural tone. The majority of cases are benign but determining which cases are potentially life-threatening or are harbingers of bad outcomes is difficult. All syncopal episodes result from the same pathophysiology; dysfunction of both cerebral hemispheres or dysfunction of the reticular activating system in the brainstem. After a syncopal episode, patients may have brief (seconds) episodes of confusion but will not experience a true post-ictal period. Although seizures are classically described as being associated with loss of urinary continence (B) and tongue biting (D) these features can also be seen in syncope. Loss of consciousness (A) is seen in generalized tonic clonic seizures as well.

What is the most common cause for syncope in a patient with a non-specific history, normal physical exam and normal ECG? Dysrhythmia Hypovolemia Idiopathic Vasovagal

Correct Answer ( C ) Explanation: Syncope is defined as a sudden temporary loss of consciousness with the inability to maintain postural tone. It is a symptom with a wide variety of life-threatening causes. Risk factors for syncope include cerebrovascular disease, cardiac manifestations, and hypertension. Despite extensive workups, no cause for syncope is found in approximately 50% of cases. When a diagnosis is made, most causes of syncope are benign and have favorable outcomes. Patients with preexisting cardiovascular disease and syncope from any cause are at the greatest short- and long-term risk of mortality. Syncope from cardiovascular causes is associated with increased mortality, whereas syncope due to neurocardiogenic, orthostatic, and medication-related syncope is not associated with increased mortality. Cardiac dysrhythmias (A) are the most common cause of cardiac syncope. Brady dysrhythmias such as heart block can lead to syncope in addition to tachydysrhythmias such as ventricular tachycardia and supraventricular tachycardia. Orthostatic hypotension, dehydration, and hypovolemia (B) can cause syncope. The most susceptible individuals are elderly patients taking blood pressure medications or diuretics. Syncope can also be the presenting symptom secondary to hemorrhage such as seen with an ectopic pregnancy. Vasovagal (D), or situational syncope, is a common cause of syncope. Multiple factors contribute to these attacks. There is typically a relatively low blood volume. Various circumstances can then lead to the syncopal event. If there is underlying fear or anxiety, the vasomotor center increases heart rate in order to increase cardiac output. However, due to a relatively low blood volume, the heart is unable to meet the requirement. The high sympathetic activity is always modulated by vagal outflow, in these cases leading to excessive slowing of heart rate. This leads to a paradoxical vagal response that causes syncope. The tilt-table test typically evokes the attack.

What nerve root is mainly responsible for the triceps reflex? C5 C6 C7 C8

Correct Answer ( C ) Explanation: The C7 nerve root is responsible for the triceps reflex. The motor function of C7 is also associated with elbow extension.

Which of the following stable patients with back pain should have an emergent MRI performed? 23-year-old man with a positive straight leg raise and otherwise normal neurologic examination 30-year-old woman with back pain and dysuria 34-year-old man with a history of intravenous drug abuse with back pain and constipation 45-year-old woman with back pain after a car accident with no midline tenderness and normal neurologic examination

Correct Answer ( C ) Explanation: The majority of patients who present with back pain do not require emergent imaging. However, those who may have a serious cause of their back pain including cauda equina syndrome, epidural abscess, vertebral osteomyelitis and other causes of cord impingement should be considered for emergent imaging of the spine. Patients that present with back pain should be screened for "red flags" in their presentations that support one of these critical diagnoses. These red flags include age (both extremes), history of cancer (especially those known to metastasize to bone), fever, weakness, saddle anesthesia, change in bowel or bladder function (incontinence or retention/constipation), trauma, history of immunocompromise (including chronic steroid use) and intravenous drug abuse. Patients with these components to their presentations have a higher risk of dangerous diagnoses and should be considered for early imaging with either CT (better for bony abnormalities) or MRI (better for spinal cord visualization) A positive straight leg raise (A) (pain radiating from the back into the leg below the knee) is suggestive of a protruding disk but can be worked up as an outpatient. Patients with symptoms of pyelonephritis (B) rarely require imaging unless there is a more complicated picture to their presentation (presence of kidney stone, sepsis, septic shock). Patients with trauma but no midline tenderness and a normal neurologic examination (D) do not require imaging.

A 66-year-old woman with atrial fibrillation on warfarin presents with dark stools for 2 days. Her vitals are T 37.7°C, HR 136, BP 81/43, RR 24, and oxygen saturation 94%. Her labs reveal a hematocrit of 19.4% (baseline 33.1%) and an INR of 6.1. Which of the following should be administered to this patient? Fresh frozen plasma only Vitamin K IM and fresh frozen plasma Vitamin K IV and fresh frozen plasma Vitamin K PO and fresh frozen plasma

Correct Answer ( C ) Explanation: The patient has a life-threatening gastrointestinal bleed in the setting of anticoagulation with warfarin, a vitamin K antagonist. Warfarin acts by inhibiting the synthesis of vitamin K-dependent factors in the coagulation cascade (II, VII, IX, X, protein C, and protein S). The anticoagulant effect of warfarin should be reversed as part of the patient's emergent treatment. Fresh frozen plasma (FFP) contains all factors in the coagulation cascade and should be given in patients with major bleeding and elevated INR. Vitamin K should be given IV in critically ill patients with elevated INR because it shortens the time to effect. Vitamin K should not be given intramuscularly (B) because absorption via this route is highly variable. Vitamin K should also not be given orally (D) in critically ill patients because the onset of action will be delayed. Additionally, absorption in patients with gastrointestinal bleeding may be variable. Vitamin K should be given along with FFP (A) because the factors inhibited rely on vitamin K for function.

A 65-year-old man presents complaining of severe right-sided frontotemporal headache, which began acutely three hours prior to arrival as he was reaching up to change a light bulb. The pain radiates through his anterior right neck and shoulder and is described as dull, throbbing, and pulsatile. The patient notes numbness and weakness of his left arm and hand. Physical exam demonstrates normal vital signs, a right-sided ptosis, a right pupil 2 mm in diameter and unreactive, and a 4-mm reactive left pupil. The patient's left upper extremity demonstrates pronator drift and diminished grip strength. Blood work is only significant for hemoglobin of 9.8 and platelet count of 96,000. What is the most appropriate treatment for this patient's condition? Consult cardiothoracic surgery for operative intervention Consult neurosurgery to perform a coiling procedure Consult neurosurgery to perform an extraventricular drain to lower intracerebral pressure (ICP) Systemic anticoagulation Systemic thrombolytic therap

Correct Answer ( D ) Explanation: The patient has a spontaneous carotid artery dissection. This uncommon cause of severe headache can be precipitated by seemingly innocuous events such as sneezing, painting a ceiling, or mild neck hyperextension. Symptoms usually include unilateral headache, neck pain, pulsatile tinnitus, and symptoms of retinal or cerebral ischemia. Horner's syndrome may develop due to the interruption of ascending sympathetic fibers in the carotid artery. The constellation of headache and neck pain and partial Horner's syndrome should prompt strong consideration of carotid artery dissection. The diagnostic study of choice is CTA or MRA of the head and neck. Systemic anticoagulation with unfractionated heparin or low molecular weight heparin is the initial treatment to prevent further stenosis and embolization. Endovascular stenting may be considered in patients with persistent or worsening neurological deficits. Consultation to cardiothoracic surgery (A) would be necessary if ascending aortic dissection were present—but this is not the case. Coiling (or angioembolization) (B) would be appropriate for a ruptured subarachnoid hemorrhage, not a vascular dissection. Lowering ICP via CSF drainage (C) has no role in the management of arterial dissection. Systemic thrombolysis (E) is critical for effective management of acute cerebral artery dissection (if the patient presents within 4.5 hours of onset) due to possible embolic and ischemic effect, but patient's must satisfy administration criteria prior to administration. In this case, patient has platelet count of less than 100k, which is a contraindication.

A 14-year-old boy presents with right eye redness for one day. Examination reveals conjunctival injection, yellow discharge, and crusting. His visual acuity is normal, intraocular pressure is 12 mm Hg, and there is no fluorescein uptake. What management is indicated? Acetazolamide Homatropine drops Polymyxin B plus trimethoprim drops Timolol drops

Correct Answer ( C ) Explanation: This patient presents with acute conjunctivitis and should be treated with a topical antibiotic. Conjunctivitis describes the presence of conjunctival inflammation manifested as eye redness and discharge. The majority of cases of conjunctivitis are viral in origin and do not require specific treatment. However, it is often difficult to differentiate between viral and bacterial conjunctivitis. Topical antibiotics are associated with benefits in patients with bacterial conjunctivitis. Patients who use topical antibiotics have higher clinical remission rates. Haemophilus influenzae, Streptococcus pneumoniae, and Staphylococcus aureus are the most common bacterial pathogens. Polymyxin B plus trimethoprim solution is the most commonly used and most extensively tested topical antibiotic. Patients with conjunctivitis should have a normal visual acuity and normal intraocular pressures (10 - 15 mm Hg).

27-year-old man presents with a severe headache and blurred vision. Physical examination reveals bitemporal hemianopsia and an inability to abduct the left eye. What diagnostic modality will most likely yield a diagnosis? CT angiography of the head CT venogram of the head MRI of the brain Noncontrast CT scan of the head

Correct Answer ( C ) Explanation: This patient presents with signs and symptoms concerning for a hemorrhage of the pituitary or pituitary apoplexy and should have an MRI for diagnosis. Pituitary apoplexy describes the presence of a pituitary infarction or hemorrhage leading to decreased pituitary function. Often, patients have a preexisting pituitary tumor. The most common symptoms seen in hemorrhagic pituitary apoplexy are sudden onset of headache caused by subarachnoid bleeding associated with nausea and vomiting. Fever may be present if the vascular supply of the hypothalamus is compromised. Patients typically have ophthalmologic symptoms including decreased visual acuity, opthalmoplegia and visual field defects. The classic visual field defect is bitemporal hemianopsia. Bleeding may cause abnormalities of cranial nerves III, IV and VI. These findings are typically due to leakage of blood into the subarachnoid space or due to mass effect (hemorrhagic or a rapidly expanding mass). Lab testing will reveal global panhypopituitary function. MRI is the optimal study for a number of reasons. The sella turcica, where the pituitary sits, is not imaged well by CT scan due to the amount of bone in the area. Additionally, MRI can differentiate between hemorrhagic and necrotic tissue, which CT cannot. Overall, MRI is 50% more sensitive for apoplexy than CT scan. CT angiography (A) is useful in the detection of arterial aneurysms. CT venogram (B) is helpful in assessing the patency of venous sinuses if cerebral venous thrombosis is suspected. A noncontrast CT of the head (D) is less sensitive than MRI for the previously stated reasons.

A 19-year-old woman is brought to the ED by paramedics for altered mental status following a motor vehicle accident. The medics report that she was not wearing her seatbelt, and she has obvious signs of trauma to her forehead. She opens her eyes when you pinch her hand. She moans when you ask her questions and pulls her hand away when you pinch it. Based on this exam, what is her Glasgow Coma Scale (GCS) score? 6 7 8 9

Correct Answer ( C ) Explanation: This patient's GCS is 8. She gets a score of 2 for eye opening, a score of 2 for the verbal response, and a score of 4 for the motor exam. The Glasgow Coma Scale was developed as a standardized scoring system to allow interobserver neurologic assessment of patients with traumatic brain injury(TBI). The GCS assesses a patient's best eye, verbal, and motor responsiveness. It was initially designed for use in hemodynamically stable and adequately oxygenated patients with isolated head trauma. However, because of its interobserver reliability, reliance on objective clinical data, and ease of application, the GCS has become a standard measure of acute neurologic function in patients with altered mental status from any cause. Answers (A, B, and D) do not correctly reflect the exam findings and GCS score. Localizing pain (5) is not recognition of where the pain is located. That's a common misconception. The patient who is able to localize pain can cross the midline and use resources on the other side of their body to remove that pain. Let's take the example of starting an IV on a confused patient. If your patient simply pulls the hand that you are starting the IV in away from you, they have withdrawn from pain (4). If they use the opposite hand to cross the midline and try to push the stimulus away, they have localized the pain. In both cases they recognize where the pain is located but only in the second example did they recognize and use resources on the opposite side of their body to cross the midline and attempt to remove the stimulus.

A 19-year-old G1P0 at six weeks by dates presents to the ED complaining of a small amount of vaginal spotting for the past two days. Vital signs are within normal limits. Your pelvic exam reveals a closed os and no adnexal tenderness or masses. Laboratory analysis reveals a serum beta hCG of 1830 mIU/ml and type B, Rh-positive blood. A transvaginal ultrasound shows an intrauterine gestational sac without a yolk sac or fetal pole and a right ovarian cyst that measures 2 x 1 cm. Given this information, which of the following is the most likely diagnosis? Blighted ovum Ectopic pregnancy Implantation bleeding Threatened miscarriage

Correct Answer ( C ) Explanation: This patient's serum beta-hCG level and transvaginal ultrasound are suspicious for a 6-week gestation; however, due to the lack of yolk sac or fetal pole, it cannot be definitely called an intrauterine pregnancy (IUP) as the diagnosis of an IUP requires a gestational sac located within the endometrium and that contains a yolk sac and/or fetal pole. Given the lack of diagnostic criteria, this bleeding should be attributed to implantation bleeding, which is relatively common in early pregnancy and occurs in around 9% of women within the first eight weeks of pregnancy. A blighted ovum (anembryonic gestation) (A) is a pregnancy in which the very early pregnancy appears normal on an ultrasound scan. But as the pregnancy progresses, a visible embryo never develops. The term blighted ovum is no longer used, it is replaced with missed abortion. In a normal pregnancy, an embryo would be visible on an ultrasound by six weeks after the last menstrual period. Because there is vaginal bleeding, the presentation is classified as a threatened miscarriage rather than a blighted ovum. The possibility of an ectopic pregnancy (B) has not been eliminated in this patient, and she requires close follow-up. However, it is less likely, given her lack of ectopic risk factors and the close approximation of her dates, beta-hCG level, and ultrasound. In the scenario, if there was an actual intrauterine pregnancy (with necessary findings including yolk sac, fetal pole and cardiac activity), the presence of 1st-trimester vaginal bleeding and a closed internal cervical os, would be classified as a threatened miscarriage (D).

A 55-year-old woman with a history of chronic bronchitis presents and is concerned about the appearance of her left eye. She denied a change in vision or any discomfort but is concerned because she noticed "blood in my eye" this morning. Evaluation of the patient demonstrates intact extraoccular muscles; pupils that are equal, round, and reactive to light; and visual acuity of 20/30 in each eye. Which of the following is the most appropriate next step in the evaluation and management of her complaint? CT scan of the orbit and facial bones to evaluate for globe injury Measure intraocular pressure to rule out a retrobulbar hematoma Obtain an ophthalmology consultation Reassure the patient and discharge her home Stain with fluorescein dye and assess for a corneal abrasion

Correct Answer ( D ) Explanation: The patient has a subconjunctival hemorrhage. This usually occurs with trivial events such as coughing or sneezing, Valsalva maneuver, or minor blunt trauma. The hemorrhage appears flat and is limited to the bulbar conjunctiva and stops abruptly at the limbus. This appearance is important to differentiate it from bloody chemosis, which can occur with scleral rupture, coagulopathy, or cavernous sinus thrombosis. Other than its appearance, this condition does not cause the patient any pain or change in visual acuity. It generally heals within 14 days. The patient in this scenario has a history of bronchitis and likely developed the subconjunctival hemorrhage from coughing. No further intervention is necessary and the patient should be reassured and discharged home. CT scan (A) is a valuable tool when there is concern for injury to the orbit or globe. It helps in the evaluation of a foreign body, orbital cellulitis, fracture, globe rupture, or retinal detachment. Intraocular pressure (B) is a valuable measurement when the differential includes acute glaucoma, hyphema, lens dislocation or swelling, suprachoroidal hemorrhage, or retrobulbar hemorrhage. Although it can be considered in individuals at risk for these conditions, it has a limited role in the diagnosis or management of subconjunctival hemorrhage. Indications for ophthalmologic consultation (C) include globe penetration, complex eyelid lacerations, penetrating foreign bodies, injuries to the anterior chamber, hyphema involving > 25% of the anterior chamber, orbital compartment syndrome, or anytime you are concerned or not entirely comfortable with the diagnosis, management, or follow-up plan. Ophthalmology consultation is not necessary for the management of a subconjunctival hemorrhage. Fluorescein staining (E) an eye helps to diagnose a corneal abrasion or ulcer. Patients with corneal injury often complain of light sensitivity with pain and a foreign-body sensation in their eye. This patient may have sustained minor trauma causing her subconjunctival hemorrhage. But absent discomfort, there is limited reason to suspect corneal injury.

A 30-year-old woman misses work and presents to the Emergency Department with severe pelvic pain rated at 6/10. She states it began yesterday with the onset of menstruation. She has regular cycles with normal blood flow amount, but has not had this pain before. She denies spine, urologic and rectal symptoms. Vital signs are normal, and physical examination as well as pelvic ultrasound is unremarkable. Serum beta-hCG is negative. Other than referral to a gynecologist for further evaluation, which of the following is the most appropriate initial treatment? Intravenous morphine Oral contraceptive pills Oral ibuprofen Oral pregabalin

Correct Answer ( C ) Explanation: This patient's working diagnosis is most concordant with primary dysmenorrhea, in which there is significant pain associated with the first few days of menses which alters normal activity or requires pain medication to control. Although secondary causes such as endometriosis and pelvic inflammatory disease are the most common misdiagnosis of primary dysmenorrhea, her stable presentation in this specialized setting, in conjunction with a normal initial diagnostic test, favors symptomatic control and follow-up evaluation. To treat dysmenorrhea, non-steroidal antiinflammatory medications (NSAIDs) and acetaminophen are helpful. Opioid analgesics (A) are considered second line if NSAIDs and acetaminophen fail to relieve the pain. Consider parenteral pain control if oral medications fail. Abdominopelvic heat is also beneficial. Contraceptive pills (B) are reserved for women who do not respond to pain medications. Pregabalin (D), a neuropathic pain medication, would not be appropriate in the acute setting, nor would it relieve the nociceptive pain of primary dysmenorrhea.

An 80-year-old man reports three months of high pitched ringing in his ears. He denies head trauma, fever, change in medications or diet, gross hearing loss, dizziness or vertigo, focal neurologic deficits or vision changes. Which of the following is the most likely diagnosis? Acoustic neuroma Ménière disease Primary tinnitus Vestibular neuritis

Correct Answer ( C ) Explanation: Tinnitus is a high-pitched noise in the ear that may last for several minutes or persist. It may be described as humming, buzzing or ringing. When symptoms are severe, it may interfere with concentration or sleep. The otologic exam is often normal. Cerumen impaction is common, and head or neck carotid bruit may be present. Tinnitus is also associated with some degree of hearing loss. Evaluation should include a review of the patients blood pressure, allergies, serum lipids and thyroid function. Since most causes are due to presbycusis (age-related, cumulative hearing damage and loss), effective therapies are limited. Medication management involves trials of diuretics, antihistamines, anticonvulsants, tricyclic antidepressants and benzodiazepines. 00:29 Tinnitus is present in patients with an acoustic neuroma (A) but they will also have symptoms of hearing loss anddisequilibirum. Acoustic neuromas, also known as vestibular schwanomas, are tumors that arise from the vestibular portion of the eighth cranial nerve. The tumors are typically unilateral. Ménière disease (B) is classically described as a tetrad of symptoms: vertigo, tinnitus, hearing loss and aural fullness. This patient only has tinnitus. Vestibular neuritis (D) is sudden, often profound, prolonged vertigo without hearing impairment in an otherwise healthy patient. Tinnitus is not associated with vestibular neuritis.

Which of the following symptoms is a hallmark of acute bronchitis? Dyspnea Frank hemoptysis Pleuritic chest pain Productive cough

Correct Answer ( D ) Explanation: Acute bronchitis is caused by inflammation of the large airways of the lung. The hallmark is productive cough. It is usually caused by respiratory viruses and is self-limited within 3 weeks. Although many clinicians prescribe antibiotics for acute bronchitis, a bacterial source is identified in less than 10% of cases. These cases are most often seen in settings of community outbreaks among college campuses and military personnel. Patients rarely experience dyspnea (A). If dyspnea is present, pneumonia should be suspected. Patients with bronchitis often have blood-streaked sputum, not frank hemoptysis (B). If hemoptysis is present, the clinician should be concerned about tuberculosis. Pleuritic chest pain (C) is more associated with pneumonia than bronchitis.

A 23-year-old woman presents with pain in the suprapubic area and right lower quadrant of her abdomen that began 4 hours ago. She denies vaginal discharge and vaginal bleeding. Her vital signs are BP 115/65, HR 60, RR 12, and T98.1°F. Her last menses was one week ago. Which of the following diagnostic tests should be obtained first? Complete blood count Gonorrhea and chlamydia DNA amplification Urinalysis Urine beta-hCG

Correct Answer ( D ) Explanation: All women of childbearing age presenting with pelvic complaints require a pregnancy test. Even though this patient reports that she had her menses one week prior to her presentation, ectopic pregnancy and even a normal pregnancy can be associated with vaginal bleeding that can be mistaken for normal menses. Since ectopic pregnancy is a life-threatening condition, it is important to rule it out early in this patient's workup. A urine beta-hCG is a rapid test that has a very high sensitivity that can rapidly determine if a woman is pregnant. A negative urine beta-hCG effectively rules out ectopic pregnancy. In an actively bleeding patient, a complete blood count (A) can provide useful information about the hemoglobin and platelet levels. A low hemoglobin level may prompt a blood transfusion. The evaluation for gonorrhea and chlamydia with DNA amplification (B) may be part of the evaluation of the patient with acute pelvic pain as a helpful adjunct in the work-up of pelvic inflammatory disease. However, in most institutions these results are not available in a timely fashion and do not have the same level of urgency as the exclusion of pregnancy with a beta-hCG test. A urinalysis (C) may also be helpful in the evaluation of a woman with pelvic pain, however, it is not entirely specific to the cause of pain. Urine may be affected by other diagnoses including cervicitis, vaginitis, pelvic inflammatory disease, nephrolithiasis, or appendicitis.

A 22-year-old man presents with a sore throat. On examination, the pharynx is erythematous without tonsillar enlargement or exudate. There is no cervical adenopathy. The patient is not sexually active. Which of the following is likely to provide the most improvement? Acetaminophen PO Amoxicillin PO Ceftriaxone IM Ibuprofen PO

Correct Answer ( D ) Explanation: The patient likely has viral pharyngitis. The examination demonstrates an erythematous pharynx without exudates and without adenopathy. In this case, the patient needs supportive care. NSAIDs, such as ibuprofen, will provide the most symptomatic relief. Acetaminophen (A) may provide some analgesic effect in this patient, but does not have the same effect as NSAIDs in improving resolution of symptoms as it lacks anti-inflammatory properties. Amoxicillin PO (B) may be helpful in the treatment of streptococcal pharyngitis although evidence does not suggest it improves time to resolution of symptoms or pain. In children, the use of antibiotics is recommended to prevent the possibility of later rheumatic heart disease. Ceftriaxone IM (C) is indicated for possible gonococcal pharyngitis, which does not have an exudative component. Clinicians cannot make the diagnosis without a careful sexual history.

A 45-year-old man presents to the ED complaining of difficulty walking. During your exam, you ask the patient to walk. You note a wide gait while his eyes are open. When you ask him to close his eyes, he is unable to walk. Which physical exam finding is expected to be abnormal? Finger-to-nose test Heel-to-shin test Rapid alternating hand movement Romberg test

Correct Answer ( D ) Explanation: Ataxia can broadly be categorized into motor and sensory ataxias. Motor ataxias are usually caused by disorders of the cerebellum. Sensory ataxias occur with failure in transmission of proprioception to the CNS. Sensory ataxias may be compensated by visual sensory information. Loss of visual information (darkness, eyes closed) leads to worsening of the sensory ataxia. The Romberg test helps to distinguish sensory from motor ataxia. The inability to maintain a steady position while standing upright with arms out and feet together confirms the presence of ataxia. If the ataxia worsens when the patient closes his or her eyes (loss of visual stimuli), then the Romberg test is positive and there is likely a sensory ataxia due to pathologic proprioceptive input (posterior column, vestibular dysfunction). Finger-to-nose (A), heel-to-shin (B), and rapid alternating hand movement (C) testing helps to distinguish pathology between the cerebellum and posterior columns (proprioception). Abnormal movements with the eyes open suggest cerebellar disease. Abnormal movements with the eyes closed suggest posterior column disorders.

A 15-year-old boy presents to the ED with an acute onset of altered level of consciousness. There is no history of trauma. He is completely unresponsive to external stimuli, including pain. Which of the following is the most likely cause of coma in this child? Brain malformation Encephalitis Inborn error of metabolism Toxic ingestion

Correct Answer ( D ) Explanation: Consciousness is a state of awareness of one's self and the environment. Coma is a state of no response to any stimuli, including pain. Alterations between these two endpoints deteriorate in a pattern of reduced awareness of self, then environment, then an absence of arousal. Confusion is a loss of clear thinking with impaired decision-making and cognition. Disorientation accompanies or follows confusion, and is characterized as being unaware of time, then place, then memory then self. Delirium follows and is marked by mental and motor excitement, irritability and agitation. Lethargy follows, where the patient is in a state of limited responsiveness, even to moderate stimulation. Stupor is marked by deep sleep or unresponsiveness, however, patients will respond to repeated moderate stimuli. Coma follows. The most common cause of acute onset of altered consciousness in the pediatric population is toxic ingestion. Congenital malformations (A) and inborn errors of metabolism (C) present most commonly in infancy and are chronic. Although encephalitis (B) is common in this age, it almost always presents with fever and is subacute in onset

An African-American man is sent home with a prescription for an antibiotic after being diagnosed with a urinary tract infection. Two days later he returns because his eyes are yellow. Which of the following antibiotics was most likely initially prescribed? Amoxicillin Cephalexin Doxycycline Trimethoprim-sulfamethoxazole

Correct Answer ( D ) Explanation: Glucose-6-phosphate dehydrogenase (G6PD) deficiency is an X-linked recessive hereditary disease that results in characteristically low levels of G6PD. This is present in 11% of African American men. Individuals with this deficiency may exhibit nonimmune hemolytic anemia in response to a variety of oxidant stressors such as exposure to particular medications or (more commonly) infection.Trimethoprim-sulfamethoxazolecan cause an oxidative stress that leads to hemolytic anemia. Other medications associated with hemolysis in G6PD deficiency include the following:

A 47-year-old man presents with hiccups for three days. He is unable to stop them with any home remedies. His physical examination is benign. Which of the following is a potential cause of hiccups? Hypercalcemia Hyperkalemia Hypoglycemia Hyponatremia

Correct Answer ( D ) Explanation: Hiccups commonly begin secondary to gastric distention resulting from a number of causes including excess eating, carbonation, and aerophagia. Additionally, excess alcohol ingestion as well as an acute emotional stress or excitement may lead to hiccups. When hiccups become intractable, evaluation for a more significant cause is indicated. Within the central nervous system, pathologies that lead to inhibition of the normal hiccup reflex include vascular malformations, meningitis, encephalitis, structural lesions. Any lesions that cause irritation to the vagus or phrenic nerves may lead to hiccups (e.g. tumors, infection, foreign bodies). Within the gastrointestinal system, there are multiple possibilities involving the stomach, esophagus and pancreas. Additionally, any lesions that potentially irritate the diaphragm (pneumonia, effusions) can cause hiccups. Myocardial ischemia or infarction is a rare cause of hiccups. In the metabolic category, hyponatremia and hypocalcemia are two possible causes of hiccups. Hypercalcemia (A), hyperkalemia (B) and hypoglycemia (C) are not common causes of hiccups.

A mother brings her full-term 3-day-old male to the ED after she noticed an episode of dark, tarry stool at home. Mom delivered the newborn vaginally with no complications. Mom is a vegan and is breastfeeding. On exam, the patient is afebrile. His abdomen is soft with no palpable masses. Rectal exam reveals dark, tarry stool that is guaiac positive. Which of the following is the most likely cause of this patient's presentation? Meckel's diverticulum Milk protein allergy Necrotizing enterocolitis Swallowed maternal blood

Correct Answer ( D ) Explanation: In the neonatal period, the most likely cause of lower gastrointestinal bleeding is swallowed maternal blood, from either delivery or cracked nipples during breast-feeding. The Apt test differentiates fetal from maternal blood. The blood in question is mixed with alkali to detect conversion of oxyhemoglobin to hematin. Fetal hemoglobin is more resistant to denaturation than adult hemoglobin is. If the supernatant stays pink after addition of alkali, the blood is fetal in origin (a positive test). This should not be confused with the Kleihauer-Betke test, used to detect fetal-maternal hemorrhage in the pregnant female. Necrotizing enterocolitis (C) is a complication of premature infants and presents with abdominal distension, bloody stools, and feeding intolerance. Milk protein allergy (B) should be suspected after introduction of a new formula. Infants who are breastfed are not immune to this condition if Mom consumes cow's milk in her diet. Since the mother in this vignette is a vegan and is breastfeeding, it is unlikely that a milk protein allergy is responsible for the patient's guaiac-positive stool. In older infants with painless hematochezia, congenital malformations such as small intestine duplication and Meckel's diverticulum (A) should be considered. Meckel's diverticulum is most common at two years of age.

A 17-year-old woman presents to the ED complaining of sore throat and cough for two days. Her review of symptoms is positive for fever, headache, and dysphagia. Vital signs are T 38.5°C, BP 110/80 mm Hg, and HR 115 bpm. Physical exam reveals bilateral tonsillar enlargement with exudates, midline uvula, and tender anterior cervical lymphadenopathy. Which of the following is the most appropriate next step in management? Aspirate the peritonsillar area using an 18-gauge needle and treat empirically with antibiotics No diagnostic testing; treat empirically with antibiotics Obtain a throat culture and have the patient follow-up in 48 hours for the results Perform a rapid antigen testing for Group A ß-hemolytic Streptococcus (GABHS) and treat with antibiotics if positive

Correct Answer ( D ) Explanation: Most cases of acute pharyngitis have a viral etiology. However, 15% of pharyngitis is caused by GABHS. The modified Centor criteria are used to predict the likelihood of streptococcal pharyngitis and assist in the decision to empirically treat or not treat with antibiotics. The patient has tonsillar exudates (+1), tender cervical lymphadenopathy (+1), and fever (+1), for a score of 3. Following the algorithm, a rapid antigen test should be obtained and, if positive, treatment with antibiotics is indicated. Peritonsillar aspiration (A) or incision and drainage is performed in patients with suspected peritonsillar abscess. This should be considered in an ill-appearing patient with fever, sore throat, trismus, and displacement of the uvula toward the unaffected tonsil. A rapid antigen test should be performed first (B and C) because if it is positive, treatment should be initiated. However, if it is negative, a culture is recommended, not treatment. Empiric antibiotics prior to rapid antigen testing or throat culture are no longer recommended.

A patient presents with ocular foreign body sensation. Examination reveals a whitish, wedge-shaped mass growing from the medial side of the left eyeball extending onto the cornea. There is no associated erythema, blisters or vesicles. Which of the following is the most likely diagnosis? Pemphigus Pinguecula Presbyopia Pterygium

Correct Answer ( D ) Explanation: Pterygium, also known as Surfer's eye, is a benign conjunctival growth that typically grows on the medial side of the sclerae usually in the palpebral fissure and extends onto the cornea. Risk factors include sunlight exposure and windy, sandy and dusty environments. Histologically, a pterygium is basically a mass of collagen degeneration and fibroblastic proliferation. Other than the visible white-colored mass, a patient with pterygium will complain of foreign body sensation, irritation, excessive tearing and dry, itchy eyes. Left untreated, it can grow over the cornea and cause scarring and visual impairment. Treatment is based on irradiation and surgical removal. Pemphigus (A) is an autoimmune skin disease typified by blistering skin. Pinguecula (B) refers to a yellowish-white deposit of degenerative calcified collagen that occurs near the limbus (the cornea-sclera junction). As compared to a pterygium, it does not appear to grow onto the cornea. Presbyopia (C), literally an "aging eye", is the age-related degeneration of the lens, which causes a loss of flexibility, and therefore difficulty visualizing proximal objects.

Which of the following is the most common sign seen in patients presenting with a pulmonary embolism? Hemoptysis Hypoxia Swollen calf Tachypnea

Correct Answer ( D ) Explanation: Tachypnea is the most common vital sign abnormality seen in acute pulmonary embolism (PE). PE is a common, life-threatening disease caused by the occlusion of pulmonary arteries by blood clots. The majority of these clots originate in the deep venous system (more commonly in the legs than in the upper body). Patients can present with various symptoms including weakness, shortness of breath, malaise, syncope, dizziness or chest pain. In addition, PE can cause changes to all of the major vital signs. Sinus tachycardia is the most common abnormality found on ECG. Hypoxia (B) and hemoptysis (A) are common in larger PE but are frequently not seen in smaller ones. While a swollen calf (C) may be indicative of a possible PE, it is not the most common finding.

A 55-year-old man presents to the emergency department complaining of palpitations, nausea, and dizziness for 30 minutes. He is afebrile, his pulse is 140 beats per minute, and his blood pressure is 78/50 mm Hg. As the nurse attaches him to the cardiac monitor, you see the rhythm strip seen above. What is the most appropriate treatment? Adenosine 6 mg IV push Defibrillation at 360 joules Procainamide 18 mg/kg infused over 30 minutes Synchronized cardioversion at 200 joules

Correct Answer ( D ) Explanation: The monitor shows a wide complex tachycardia. Clinically, he is hypotensive and unstable, which necessitates the use of electrical rather than pharmacologic treatment. Synchronized cardioversion is indicated for the treatment of unstable tachydysrhythmias, including certain supraventricular dysrhythmias as well as monomorphic ventricular tachycardia. For treatment of ventricular fibrillation or polymorphic ventricular tachycardia, defibrillation (not synchronized cardioversion) is indicated. Adenosine (A) is a short-acting AV-nodal blocker used frequently in supraventricular dysrhythmias. Its use is not recommended in hypotensive patients with a wide-complex tachycardia. Defibrillating a patient who has a pulse (B) is dangerous and can result in the R-on-T phenomenon. This occurs when a depolarizing impulse (endogenous or exogenous) is delivered during ventricular repolarization (T wave). This can result in polymorphic ventricular tachycardia or ventricular fibrillation. To avoid this complication, synchronized cardioversion coordinates delivery of an electrical impulse so that it occurs with initiation of ventricular contraction (i.e., at the beginning of the QRS complex), thereby avoiding a shock during the relative refractory period of the cardiac cycle. Procainamide (C) is an option for patients with stable ventricular tachycardia, but it is not appropriate treatment of a hypotensive patient with an undifferentiated wide complex tachycardia. Procainamide may also worsen hypotension.

13-year-old boy presents because his mother noticed a progressive left-sided limp over the previous three weeks. The patient states that he has pain in his knee. On exam, you elicit pain on internal rotation of the hip. Which of the following is the most likely diagnosis? ALegg-Calvé-Perthes disease BOsteosarcoma CSeptic arthritis DSlipped capital femoral epiphysis

Correct Answer ( D ) Explanation: The patient has a slipped capital femoral epiphysis (SCFE). SCFE is a Salter-Harris type 1 fracture of the femoral capital epiphysis. The x-ray will show the classic posterior and inferior displacement of the head (capital) of the femur. SCFE occurs in boys twice as often as girls, is commonly seen in children >10 years of age, and classically presents in obese children as subacute hip pain with a limp. Some underlying metabolic disorders such as hypothyroidism, panhypopituitarism, and hypogonadism are also risk factors for development of SCFE. Legg-Calvé-Perthes disease (A) is avascular necrosis of the femoral head, usually seen in young school-aged children. Hip pain and limp are early signs; radiographs may be normal. It occurs bilaterally in 15% of cases. Malignancy (B) could also present with subacute symptoms. Most osteosarcomas arise as solitary lesions within the fastest growing areas of the long bones of children. The top three affected areas are the distal femur, the proximal tibia, and the proximal humerus, but virtually any bone can be involved. The most common presenting symptom of osteosarcoma is pain, particularly with activity. Septic arthritis (C) is the most common cause of painful hip in infants and typically presents as an acute monoarthritis in an ill-appearing baby.

A 15-year-old female presents with generalized abdominopelvic pain which occurs every month after her regular, nonpainful menses. The pain, which is associated with headaches, bloating and depressed mood, begins 18 days after the last day of menstruation. She also complains of cyclic ankle swelling but denies a history of increased salt intake. Which of the following is the most likely diagnosis? Amenorrhea Dysfunctional uterine bleeding Dysmenorrhea Premenstrual syndrome

Correct Answer ( D ) Explanation: This patient displays some of the common symptoms of premenstrual syndrome, a poorly understood condition of physical, mood and behavioral changes which occur during the second half of the typical 28-day menstrual cycle. On average, menstrual flow lasts 4-5 days, beginning on day 1 of the menstrual cycle (the beginning of the follicular phase), and ending on day 4 or 5. Around day 14, ovulation occurs, and the second half of the cycle, the luteal phase, begins. If the oocyte is not fertilized, the luteal phase, on average, ends around day 28. Subsequently, a new cycle commences with the onset of another menses. The symptoms of premenstrual syndrome occur in this second half period (day 14-28), most commonly occurring on day 23-27, which is typically 18-19 days after the last day of the most recent menstruation. Amenorrhea (A) is defined as the absence of menstrual bleeding for >3 cycles. This patient has regular menses, as such, amenorrhea is incorrect. Dysfunctional uterine bleeding (B) is defined as excessive and noncyclic menstrual flow which is due to a nonpathological source. Dysmenorrhea (C) is defined as function-altering pain during menstrual flow which necessitates medical attention, namely for pain control.

A 9-year-old girl presents with scalp itching. Physical examination reveals the finding seen in the image above. What is the first-line treatment for this disease? Clotrimazole Ivermectin PO Lindane Permethrin

Correct Answer ( D ) Explanation: This patient presents with pediculosis capitis or head lice and should be treated with permethrin. Pediculosis capitis is caused by lice infection. These parasitic lice infest and lay eggs at the base of the hair shaft. Transmission is from person-to-person. It is common in children but uncommon after puberty. Patients will present with intense pruritus, which coincides with the lice feeding and inspection reveals nits (immature lice) firmly attached to the base of hair shafts. The heaviest infection is typically seen behind the ears. Diagnosis is made on clinical grounds or with microscopy. The preferred treatment for pediculosis capitis is with permethrin, which kills the adult louse. Subsequently, the nits must be removed with a vinegar solution and fine-tooth comb. The scalp should be reexamined one week later and repeat therapy at this point may be required. Clotrimazole (A) is an antifungal medication that does not play a role in the treatment of pediculosis. Ivermectin PO (B) is a second-line therapy for pediculosis capitis and should be reserved for cases in which topical therapy fails. Ivermectin is given as a single dose and repeated 10 days later but it is a second line treatment. There is also a topical ivermectin that can be used first line. In the past, lindane (C) was indicated as first line treatment but now is reserved for treatment failures.

19-year-old woman with no past medical history presents with an acute onset of headache that began eight hours ago. The headache was severe at onset and associated with nausea and vomiting. Which of the following statements is true? A normal noncontrast head CT rules out subarachnoid hemorrhage Opiates are first line medications for headaches Resolution of symptoms with metoclopramide is diagnostic of a migraine Severe anemia may create a false negative noncontrast head CT

Correct Answer ( D ) Explanation: This patient presents with symptoms concerning for a subarachnoid hemorrhage (SAH). Noncontrast head CT sensitivity and specificity have increased markedly over the years but a false negative can occur in patients with significant anemia. Anemia affects the density of blood as it is seen on noncontrast head CT and makes it difficult to distinguish blood from surrounding brain tissue. SAH represents one of the critical, life-threatening diagnoses for headache presentations. It typically presents with an acute onset of severe headache (traditionally referred to as the "thunderclap" headache). SAH often results from an underlying arterial aneurysm and making the diagnosis of SAH early in the patient's clinical course can lead to decreased morbidity and mortality. Patients suspected of SAH should be worked up with a noncontrast head CT and, if negative, a lumbar puncture as it is the most sensitive and specific test for this diagnosis.

Which of the following conditions is associated with ascending weakness? Botulism Ciguatera poisoning Miller-Fisher variant of Guillain-Barré syndrome Tick paralysis

Correct Answer ( D ) Explanation: Tick paralysis usually occurs in the spring and summer months, and most often in young children, especially girls because their long hair conceals the embedded tick. Tick paralysis occurs when the female tick attaches to a host and releases a neurotoxin that produces cerebellar dysfunction or ascending paralysis. Symptoms begin 4 -7 days after the tick attaches. Initial manifestations include restlessness and irritability, followed by ascending paralysis flaccid, acute ataxia, or a combination of the two. Deep tendon reflexes are usually lost. Treatment consists of simply removing the tick. Improvement is seen typically within a few hours and complete recovery within 48 hours. Botulinum toxin (A) binds to the presynaptic terminus of neuromuscular and autonomic synapses resulting in a decrease of acetylcholine release. Flaccid paralytic symptoms ensue, first in a bulbar distribution then descending. Ciguatera poisoning (B) is associated with ingestion of ciguatoxin, a substance with anticholinesterase and cholinergic properties that lowers the threshold for opening of voltage-gated sodium channels. The latter results in depolarization and heightened neurologic response with symptoms that include dysesthesias and paresthesias around the throat and perioral area, a burning sensation in the feet, and distortion of temperature sensation. The Miller-Fisher variant of Guillain Barré syndrome (C) is associated with a descending paralysis. It usually affects the eye muscles first and presents with the triad of ophthalmoplegia, ataxia, and areflexia. The ataxia predominantly affects the gait and trunk, with the limbs relatively spared.


Ensembles d'études connexes

Final Test Organizational Behavior Questions

View Set

Chapter 10: Divorce, Remarriage, and Blended Families

View Set

First English class/первое занятие по английскому языку

View Set

TAXES: TAXABLE INCOME/CORP + GOV SECURITIES

View Set

management control systems chapter 5: Profit Centers

View Set